You are on page 1of 362

QMI1500

EXAM PACK
UNIVERSITY EXAMINATIONS

mm
mm

MAY/JUNE 2020

QMI1500
ELEMENTARY QUANTITATIVE METHODS
Duration: 2 Hours 30 Minutes 100 Marks
EXAMINERS
FIRST MR MB NZUZA
SECOND PROF CJ SWANEPOEL
EXTERNAL DR HZ WIGGINS (University of Pretoria)

This paper consists of 12 pages, which include a list of formulas on page 12.
Programmable calculator permissible.
Instructions:
The paper consists of 30 questions for a total of 100 marks. Answer all the questions.
Only one option indicated as [1], [2], [3] or [4] per question is correct.
Marks will not be deducted for incorrect answers.
The answers to the examination MCQ may only be submitted online.

Submit the answers on my Unisa by

1. On the landing page for my Unisa, before login, go back to the link where you downloaded your examination
paper: Login and download my Exam Question Paper for May/June 2020.
2. Login using your student number and my Unisa password.
3. On the next screen, find the module code QMI1500. Click on the link to “submit MCQ”. This link will only
display if the examination session is still open for submissions.
4. A new screen will open that will guide you through the steps to upload your answers.
5. Enter the total number of questions (30) for the paper in the Number of Questions field.
6. Read the Honesty Declaration statement, then click the check box to acknowledge that you have read
the statement.
7. If you agree with the Honesty Declaration statement, type I AGREE in the text box. You cannot continue
with the submission process if you do not complete the requirements of the declaration.
8. When done, click on the Continue button.
9. You will now be able to fill out the multiple-choice question answers. When done, click on the Continue
button. A screen presenting a summary of all your answers will be displayed. Use it as a final check.
10. Click on the Continue button to submit your assessment. If you do not click Continue, no submission action
will take place.
11. A submission report will be displayed. This is your proof that your answers were successfully submitted.
If you do not see this screen, Unisa has not received your submission. Print this page or make a screen
capture for record purposes.

If you fail to submit after following the above instructions, submit via the following alternative link:
https://tinyurl.com/ya6cw8df.

If you are unsuccessful with the alternative submission link, send error reports to the primary lecturer. Only
emails from the official my Life addresses will be considered.

[TURN OVER]
CONFIDENTIAL QMI1500
Page 2 of 12 May/June 2020
Question 1
The expression
7 3 1 1
2 × ÷3 +3
9 4 8 3
is equal to
1 62
[1] 3 + .
3 75
10 2
[2] 3
+ .
3
10 5
[3] +5 .
3 24
2 5
[4] × .
31 1

Question 2
Simplify
2 1
x3 × x5
1 .
x2
47
[1] x 30
11
[2] x3
11
[3] x 30
22
[4] x 30

Question 3
Eight men and five women have volunteered to serve on a committee. How many different committees
can be formed containing four men and four women?
[1] 52
[2] 1 287
[3] 104
[4] 350

Question 4
The following observations are arranged in ascending order:
17; q; 24; q + 7; 35; 36; 46.

The median of the data is 25. The value of q, rounded to the nearest integer, is
[1] 4.
[2] 18.
[3] 25.
[4] 32.

[TURN OVER]
CONFIDENTIAL QMI1500
Page 3 of 12 May/June 2020

Questions 5 and 6 are based on the following:


A restaurant collects soft drink cans for recycling. The number of cans collected in two weeks are given
below:
84; 97; 77; 31; 84; 63; 58; 71; 47; 84; 69; 95; 43; 63.
Give the answer to the questions correct to the nearest integer.
Question 5
The mean number of soft drink cans collected per day is
[1] 63.
[2] 84.
[3] 70.
[4] 69.

Question 6
The mode of the number of soft drink cans for the two weeks is
[1] 69.
[2] 84.
[3] 70.
[4] 66.

Question 7
Consider a data set with a mean of 8,5 and variance of 16. The coefficient of variation for this dataset,
rounded to two decimal places, is
[1] 0,85.
[2] 0,16.
[3] 0,47.
[4] 0,53.

Question 8
Francinah wants to buy a smartwatch online. She finds a supplier in London selling the watch for
£510,20. This supplier is offering free shipping on the product. She then finds the same watch from a
supplier in New York for $552,10 with a shipping fee of $2,80. Next she looks up the exchange rates to
see which supplier has the better deal:
$1,00 = R17,85
£1,00 = R19,56
The total cost, in Rand, for the watch is
[1] R9 904,97 if she buys from the New York supplier.
[2] R10 034,24 if she buys from the London supplier.
[3] R9 857,79 if she buys from the New York supplier.
[4] R9 107,07 if she buys from the London supplier.

[TURN OVER]
CONFIDENTIAL QMI1500
Page 4 of 12 May/June 2020

Questions 9 to 11 are based on the following information:


The company, Model Traders, wants to construct indexes of the prices and quantities of its three prod-
ucts. The prices and quantities sold for the years 2015 and 2019 are shown below:

Product prices Quantities sold


(in Rand) (in Rand)
Product 2015 2019 2015 2019
X 40 50 1 200 1 500
Y 30 36 1 500 2 000
Z 20 24 2 000 2 700

Question 9
The simple price index for product Y for 2019 with 2015 as the base year, rounded to two decimal places,
is
[1] 133,33.
[2] 125,00.
[3] 120,00.
[4] 83,33.

Question 10
The Paasche price index for 2019 with 2015 as the base year, rounded to two decimal places, is
[1] 159,25.
[2] 130,83.
[3] 130,74.
[4] 121,72.

Question 11
The quantity
40 × 1 500 + 30 × 2 000 + 20 × 2 700
× 100
40 × 1 200 + 30 × 1 500 + 20 × 2 000
is
[1] the Paasche price index for 2019 with 2015 as the base year.
[2] the Paasche quantity index for 2019 with 2015 as the base year.
[3] the Laspeyres price index for 2019 with 2015 as the base year.
[4] the Laspeyres quantity index for 2019 with 2015 as the base year.

[TURN OVER]
CONFIDENTIAL QMI1500
Page 5 of 12 May/June 2020

Question 12
If you spend R600 on petrol for travelling 250 km driving at a constant speed, about how much would
you spend for travelling 450 km at the same speed?
[1] R1 050
[2] R200
[3] R1 080
[4] R5 100

Question 13
 
A rectangular plot of land has sides that are 1 200 m long and 900 m wide. If a hectare 10 000 m 2 of
the land costs R5 200,00, the cost of the entire plot is
[1] R5 616 000,00.
[2] R2 076 923,08.
[3] R561 600,00.
[4] R207 692,31.

Question 14
A newly released book has sold 500 copies per month when priced at R150, and 450 copies per month
when priced at R200. Let x represent the price of the book and let y represent the number of copies
sold per month. Assume that the relationship between price and sales is linear, and that the sales are
dependent on the price of the book. The linear equation you could use to predict sales for other prices is
[1] y = x + 650.
[2] y = −x + 650.
[3] y = 150x + 500.
[4] y = 200x + 450.

Question 15
The type of sampling that gives each population unit the same chance of being selected is
[1] systematic sampling.
[2] stratified sampling.
[3] simple random sampling.
[4] none of the above.

[TURN OVER]
CONFIDENTIAL QMI1500
Page 6 of 12 May/June 2020

Question 16
The parabola that is described by the quadratic equation
y + 4x = 3 + 2x2

is represented by one of the four graphs below. Calculate the vertex of the parabola. Using this
information, the correct graph is
[1] [2]
y y

5 5

4 4

3 3

2 2

1 1

x x
−3 −2 −1 1 −1 1 2 3
−1 −1

[3] [4]
y y

5 5

4 4

3 3

2 2

1 1

x x
−1 1 2 3 −1 1 2 3
−1 −1

[TURN OVER]
CONFIDENTIAL QMI1500
Page 7 of 12 May/June 2020

Question 17
The x-intercepts of the parabola that is described by the quadratic equation
2x2 + 10 = 12x − y

are
[1] (1; 0) and (5; 0).
[2] (0; −10) and (3; 8).
[3] (1; 0) and (3; 8).
[4] (−1; 0) and (−5; 0).

Question 18
The function
f (x) = kx
is strictly decreasing with an increase in x if
[1] k > 1.
[2] k = 1.
[3] k = 0.
[4] 0 < k < 1.

Question 19
Consider the system of equations
2s + 3t = 12
5s = t + 13.

When solving this system of equations simultaneously, the result is


[1] s = −3 and t = −2.
[2] s = 6 and t = 0.
[3] s = 1,5 and t = 2.
[4] s = 3 and t = 2.

[TURN OVER]
CONFIDENTIAL QMI1500
Page 8 of 12 May/June 2020

Question 20
The correct graphical representation of the solution space of the system of linear inequalities
−4x + 3y > −12
y ≤2
2x + 3y ≤ 6

is
y y
[1] [2]
6 6

5 5

4 4

3 3

2 2

1 1

x x
−4 −3 −2 −1 1 2 3 4 5 6 7 −4 −3 −2 −1 1 2 3 4 5 6 7
−1 −1

−2 −2

−3 −3

y y
[3] [4]
6 6

5 5

4 4

3 3

2 2

1 1

x x
−4 −3 −2 −1 1 2 3 4 5 6 7 −4 −3 −2 −1 1 2 3 4 5 6 7
−1 −1

−2 −2

−3 −3

[TURN OVER]
CONFIDENTIAL QMI1500
Page 9 of 12 May/June 2020

Question 21
A gym offers your first session of every month free of charge and thereafter charges you R35 per session.
Recently, the gym has added another option that allows you to pay a fixed amount of R100 per month
plus a fee of R20 for each session.
Let x represent the number of gym sessions per month.
Let y represent the total cost per month.
The total monthly cost for your gym sessions depends on the number of times you visit the gym per
month. The system of linear equations that will help you decide which option suits you the best, is
[1] y = (35 + 20)x and
y = 100 − x.
[2] y = 35(x − 1) and
y = 100 + 20x.
[3] y = 35x − 1 and
y = (100 + 20)x.
[4] y = 35x + 100 and
y = 20(x − 1).

Question 22
Solving for x in the linear inequality
4x + 4 < 6x − 8
gives
[1] x < 7.
[2] x > 6.
[3] x < 6.
[4] x > 12.

Question 23
Consider the inequality
4x + 6y + 3z ≤ 120.
The set(s) of values for x, y and z that satisfy the above inequality is/are
(i) x = 9, y = 7, z = 0.

(ii) x = 25, y = 8, z = 1.

(iii) x = 9, y = 0, z = 18.

[1] only (i) and (ii)


[2] only (ii)
[3] only (ii) and (iii)
[4] only (i) and (iii)

[TURN OVER]
CONFIDENTIAL QMI1500
Page 10 of 12 May/June 2020

Question 24
The profit (in rand) from selling x products is given by the function
P (x) = −0,01x2 + 145x − 11 000.

Use the marginal profit function to determine the number of products that have to be sold in order to
maximise the profit. The correct answer is
[1] 7 250 products.
[2] 514 625 products.
[3] 11 000 products.
[4] 14 424 products.

Question 25
You invest R1 500,00 at a simple interest rate of 8% per annum for six and a half years. What is the
total interest that you receive?
[1] R720,00
[2] R2 280,00
[3] R780,00
[4] R2 220,00

Question 26
Louis has an obligation to pay a sum of R7 000,00 in four years from now. His creditor permits him to
discharge the debt by paying R2 000,00 in one year from now and a final payment of X in three years
from now. The interest rate charged on all amounts is 11% per year, compounded quarterly. This is
represented by the following time line with the debt above the line and the payments below the line:
R7 000

0 1 2 3 4 years
R2 000 X
@ 11% per year, compounded quarterly
The value of X is equal to
[1] R3 795,40.
[2] R3 784,33.
[3] R3 842,11.
[4] R4 485,83.

[TURN OVER]
CONFIDENTIAL QMI1500
Page 11 of 12 May/June 2020

Question 27
How much should we invest now at 9% interest rate if we wish to have the total amount of R350 000,00
accumulated in five years, compounded monthly?
[1] R223 544,89
[2] R31 500,36
[3] R221 563,55
[4] R189 362,23

Question 28
Determine the future value of an annuity after ten monthly payments of R600,00 at an interest rate of
12% per annum, compounded monthly.
[1] R5 000,00
[2] R15 529,24
[3] R672,00
[4] R6 277,33

Question 29
Determine the present value of an annuity with quarterly payments of R800,00 compounded quarterly
at an interest rate of 12,5% per annum and with a term of eight years.
[1] R25 631,35
[2] R16 037,02
[3] R16 532,25
[4] R25 586,33

Question 30
Suppose you purchase vacant land for R450 000,00 with a down payment (deposit) of R55 000,00. You
secure a loan from a bank for the balance at 10,5% per annum, compounded monthly, with a term of 15
years. What are the monthly payments?
[1] R25 635,21
[2] R55 563,15
[3] R4 366,33
[4] R57 752,36

[TURN OVER]
CONFIDENTIAL QMI1500
Page 12 of 12 May/June 2020

FORMULAE
n
1
I = P RT S =P +I x= xi
n i=1

n

(xi − x)2
S i=1
S = P (1 + RT ) P = S2 = n−1
(1 + RT )

Q3 − Q1
D = SdT P =S−D QD =
2
P S
P = S(1 − dT ) S= CV =
(1 − dT ) x

S m!
S = P (1 + R)T P = m Px =
(1 + R)T (m − x)!
 
ln PS  1
S T m!
T = R= −1 m Cx =
ln(1 + R) P (m − x)!x!

⎡ ⎤
 n
    1
(1 + i) − 1 i ⎢ GDPn n ⎥
S =R× R=S× ⎣ − 1⎦ × 100
i (1 + i)n − 1 GDPo

   
(1 + i)n − 1 i(1 + i)n
P =R× R=P×
i(1 + i)n (1 + i)n − 1

y = ax + b y = ax2 + bx + c discriminant = b2 − 4ac



y2 − y1 b b b2 − 4ac
a= xm = − x=− ±
x2 − x1 2a 2a 2a
 
pn qo pn qn
PL (n) =  × 100 QP (n) =  × 100
po qo pn qo
 
pn qn po qn
PP (n) =  × 100 QL (n) =  × 100
po qn po qo

Pn pn qn
In = × 100 V =  × 100
Po po qo

If y = ax (a > 0, a = 1), then loga y = x.


If f (x) = a (a is a constant), then f  (x) = 0.

If f (x) = xn , then f  (x) = nxn−1 .

If f (x) = axn , then f  (x) = anxn−1 .

If f (x) = g(x) + h(x), then f  (x) = g  (x) + h (x).

©
UNISA 2020
ROYAL ACADEMY TUTORS
Empowering you to conquer your world

Providing World Class Tuition Fit for Kings & Queens


Contact Kingston on 083 427 5621 for Online Classes/Exam Preparation Page 2
ROYAL ACADEMY TUTORS
Empowering you to conquer your world

Providing World Class Tuition Fit for Kings & Queens


Contact Kingston on 083 427 5621 for Online Classes/Exam Preparation Page 3
ROYAL ACADEMY TUTORS
Empowering you to conquer your world

Providing World Class Tuition Fit for Kings & Queens


Contact Kingston on 083 427 5621 for Online Classes/Exam Preparation Page 4
ROYAL ACADEMY TUTORS
Empowering you to conquer your world

Providing World Class Tuition Fit for Kings & Queens


Contact Kingston on 083 427 5621 for Online Classes/Exam Preparation Page 5
ROYAL ACADEMY TUTORS
Empowering you to conquer your world

Providing World Class Tuition Fit for Kings & Queens


Contact Kingston on 083 427 5621 for Online Classes/Exam Preparation Page 6
ROYAL ACADEMY TUTORS
Empowering you to conquer your world

Providing World Class Tuition Fit for Kings & Queens


Contact Kingston on 083 427 5621 for Online Classes/Exam Preparation Page 7
ROYAL ACADEMY TUTORS
Empowering you to conquer your world

Providing World Class Tuition Fit for Kings & Queens


Contact Kingston on 083 427 5621 for Online Classes/Exam Preparation Page 8
ROYAL ACADEMY TUTORS
Empowering you to conquer your world

Providing World Class Tuition Fit for Kings & Queens


Contact Kingston on 083 427 5621 for Online Classes/Exam Preparation Page 9
ROYAL ACADEMY TUTORS
Empowering you to conquer your world
After plotting Q20 on the Geogebra App, it looks like the diagram below.
The correct graphical representation is shown by the darkest region

Providing World Class Tuition Fit for Kings & Queens


Contact Kingston on 083 427 5621 for Online Classes/Exam Preparation Page 10
ROYAL ACADEMY TUTORS
Empowering you to conquer your world

Providing World Class Tuition Fit for Kings & Queens


Contact Kingston on 083 427 5621 for Online Classes/Exam Preparation Page 11
ROYAL ACADEMY TUTORS
Empowering you to conquer your world

Providing World Class Tuition Fit for Kings & Queens


Contact Kingston on 083 427 5621 for Online Classes/Exam Preparation Page 12
ROYAL ACADEMY TUTORS
Empowering you to conquer your world

Providing World Class Tuition Fit for Kings & Queens


Contact Kingston on 083 427 5621 for Online Classes/Exam Preparation Page 13
ROYAL ACADEMY TUTORS
Empowering you to conquer your world

Providing World Class Tuition Fit for Kings & Queens


Contact Kingston on 083 427 5621 for Online Classes/Exam Preparation Page 14
ROYAL ACADEMY TUTORS
Empowering you to conquer your world

Providing World Class Tuition Fit for Kings & Queens


Contact Kingston on 083 427 5621 for Online Classes/Exam Preparation Page 15
ROYAL ACADEMY TUTORS
Empowering you to conquer your world

Providing World Class Tuition Fit for Kings & Queens


Contact Kingston on 083 427 5621 for Online Classes/Exam Preparation Page 16
10mm by 20mm

1:250
,
,
,
,
, , , ,
, , , ,

, , , ,
, , , ,

, , , ,
, , , ,

, , , ,
, , , ,

,
,
,
,
FBN1502
May/Jun 2015

Question 31
26
Match each inequality with its graph.
1 1
nnnnnnnnnnn1. 2y + x ≤ 6 2. x − y > 4 3. y < 3 + x 4. 4y + 2x ≥ 16
2 2
a. b.
y y

1 4

x 3
−1 1 2 3 4 5 6 7 8 9
−1 2

−2 1

−3 x
−1 1 2 3 4 5 6 7 8 9
−4 −1

c. d.
y y

4 4

3 3

2 2

1 1

x x
−1 1 2 3 4 5 6 7 8 9 −8 −7 −6 −5 −4 −3 −2 −1
−1 −1

The correct answer is

[1] [2]
Inequality Graph Inequality Graph
1 c 1 b
2 a 2 d
3 d 3 c
4 b 4 a
[3] [4]
Inequality Graph Inequality Graph
1 a 1 b
2 d 2 c
3 b 3 a
4 c 4 d
[5] none of the above.

Page 12 of 19
FBN1502
May/Jun 2015

Question 32
27
Consider the following graph:

x
−5 −4 −3 −2 −1 1 2 3 4 5
−1

−2

−3

−4

The system of inequalities whose solutions space is represented by the shaded region, is

[1] [2]
−4x + 3y < −12 −4x + 3y ≥ −12
y ≥ 2 y > 2
2x + 3y > 6 2x + 3y > 6

[3] [4]
−4x + 3y > −12 −4x + 3y > −12
y ≥ 2 y ≤ 2
2x + 3y > 6 2x + 3y < 6

[5] none of the above.

Page 13 of 19
QMI1500 October 2019 Memo
Question Answer
1 1
2 2
3 1
4 1
5 4
6 2
7 3
8 2
9 3
10 3
11 2
12 1
13 4
14 1
15 4
16 1
17 4
18 1
19 1
20 1
21 3
22 3
23 1
24 1
25 3
26 4
27 2
28 3
29 2
30 3
Question 1
'
If x = 4, find the value of the sum &()(5 + 𝑥&)

Solution

• Substitute x = 4 into this equation: '&()(5 + 4& )


• When t = 0, you get (5 + 4) ) = 5 + 1) = 5 + 1 = 5
• When t = 1, you get (5 + 4- ) = 5 + 4 = 5 + 2 = 7
• When t= 2, you get (5 + 4. ) = 5 + 16 = 5 + 4 = 9
• When t= 3, you get (5 + 4' ) = 5 + 64 = 5 + 8 = 13
• The sum of all these gives 5 + 7 + 9 + 13 = 34

Question 2

Solve for x in the following equation


'01. - -130
- =
2 2 .

Solution
'01.1- '01'
• Left Hand Side gives: =
2 2
-130
• Right Hand Side gives:
.
.∗(-130)
o Multiplying top and bottom by 2 gives Right Hand Side gives: =
.∗.
.1-)0
2
'01' .1-)0
Now equate the left to the right and you get: =
2 2

Since the denominators are equal, it means that we can also equate the numerators. This gives
3x -3 = 2 -10x

Manipulate the equation to get 13x = 5


3
And therefore x =
-'

Question 3

Find r if 4𝑟 + 3 − 2 = 3

Solution

Manipulate the equation to isolate the variable that you are trying to solve for

4𝑟 + 3 = 3 + 2 = 5
.
Square both sides to give: 4𝑟 + 3 = 5. = 25
On the left: the square root and the power of 2 cancel each other out. We are left with just 4r
+3

Equating both sides gives 4r + 3 = 25

More manipulation gives: 4r = 25 – 3 = 22


.. --
Divide both sides by 4 and get r = =
2 .

Question 4

Find the value of the following expression:


3 - -
𝟒 - 3 + 1
; 2 3

Solution

• Convert the mixed fractions into improper fractions (i.e. where the numerator is
bigger than the denominator)
3 '<
o 𝟒 becomes
; ;
- -'
o 3 becomes
2 2
- =
o 1 becomes
3 3
• Find common denominator
o 8 and 4 already have a common denominator between them which is 8.
Multiply this by 5 to get a common denominator amongst all 3 – you get 40
• Adjust numerators accordingly before adding them
o Multiply 37 by 5, to get 185
o Multiply 13 by 10, to get 130
o Multiply 6 by 6, to get 48
o Adding these up gives a numerator of 103 (185 – 130 + 48)
-)'
• The new expression of
2)
.'
• Simplifying this gives us 2
2)

Question 5

Simplify the following:

80 ∗ 8'0
2'0 ∗ 40?.
Solution

Denominator

• need the same base before you can add exponents


• tip: 4 = 22
• therefore: 4x+2 = (22)x+2 = 22x+4
• now that you have the same base, you can add the exponents (i.e. 3x + 2x + 4 = 5x
+4)
• the denominator is therefore equals 25x+4
Numerator

• same base, therefore can add exponents (i.e. add x and 3x to give 4x)
• this gives 820
• we know that 8 = 2*2*2 = 23
• therefore 84x = (23)4x = 212x

So you now have the following

2-.0
230?2
This can be further simplified since you have the same base

• Tip: division means you have to subtract the exponents


• 12x – (5x + 4) = 12x -5x – 4 = 7x – 4

So you now have the following

2<012
This can be re-written as: 27x * 2-4

Remember that a negative exponent means the inverse function


-
• Therefore 2-4 =
.@

.AB
You now have:
.@

.AB
Since 24 = 2*2*2*2 = 16, this gives a final answer of
-=

Question 6

Simplify the following expression:


. .
𝑥3 ∗ 𝑥'
.
𝑥 12
Solution

Denominator
. -
• =
2 .
• Therefore x-2/4 = x-1/2
-
• This is the same as C
0D

Numerator

• Same base therefore can add exponents


• Must find common denominator first
• Tip – easiest is to multiply the two numbers 3 and 5 to give you 15 as the common
denominator
• Adjust the numerators before you add them (i.e. multiply 2 by 3 to get 6; and multiply
the other 2 by 5 to get 11). Adding 6 and 10 gives 16
• Therefore 2/5 + 2/3 = 16/15
• x2/5 * x2/3 = x16/15
Putting these together gives the following:
-=
𝑥 -3
-
𝑥 1.
Because we are dividing where the base is the same we can subtract the exponents,
-= - -= -
giving ( – (− )). This is the same as +
-3 . 3 .

once again, we need a common denominator, which is 30 (multiplying 15 and 2)

we then adjust the numerators before adding them (multiply 16 by 2 giving 32 and multiply 1
by 15 giving 15). Adding 32 and 15 gives us 47

so we have x47/30
2<
is an improper fraction. This can therefore be further simplified by writing the exponent as
')
-<
a mixed number gives 1
')

Question 7

Denzel buys 300 outdoor refuse bags for R300 (VAT included). He sells them for R25.99 for
a pack of 20. How much profit did he make?

Solution

This means each bag cost him R1. Therefore a pack of 20 would have cost him R20 (20 * R1)
'))
How many bags of 20 in a total of 300 bags? = = 15 bags of 20
.)

He sells the bag of 20 for R25.99. The difference between cost price and revenue received is
R5.99 (R25.99 – R20)
Profit per bag of 20 is worked out to be R5.99

Profit on all 15 bags of 20 is then R89.85 (R5.99 * 15)

Question 8

Consider the below to be the monthly income of 10 drivers. Find the following:

• Average income
• Median of the incomes
• Mode of the incomes
• The standard deviation
• The quartile deviation
1 080.00
2 000.00
1 580.00
1 540.00
2 500.00
1 800.00
1 580.00
3 000.00
3 280.00
2 930.00

Solution

The variable x is the income, where x1 = 1 080, x2 = 2 000…..x10 = 2 930

X can take on 10 different values, therefore n= 10

To find the average, use the formula


G -)
1 1
x = x𝑖
n 10
H(- H(-

-
This gives (1080 + 2000 +1580 + 1540 +2500 + 1800 +1580 + 3000 + 3280 + 2930)
-)

-
= (21 290)
-)

= 2 129

To get the median, you need to sort the data, giving the below list.
1 080.00
1 540.00
1 580.00
1 580.00
1 800.00
2 000.00
2 500.00
2 930.00
3 000.00
3 280.00

There are 2 middle values (1800 and 2000). Therefore you need to take the average of the
- .
two to be the median. Using the formula for arithmetic mean you should get
. H(- x 𝑖

- -
The median is therefore (1800 + 2000) = (3800) = 1 900
. .

The mode is the number that occurs most often. In this case that income is 1 580

The variance is derived from the formula


-
1 .
xi − 𝑥
n−1
-)

We have already solved for the arithmetic mean to be 2129, so we get:

-
1 .
xi − 2129
9
-)

-
= [ (1080 – 2129)2 + (1540 – 2129)2 + (1580-2129)2 + (1580 – 2129)2 + (1800 – 2129)2 +
M
(2000 – 2129)2 + (2500 – 2129)2 + (2930 – 2129)2 + (3000 – 2129)2 + (3280 – 2129)2]
-
= [(-1049)2 +(-589)2 + (-549)2 + (-549)2 (-329)2 + (-129)2 + (371)2 + (801)2 + (871)2 +
M
(1151)2]
-
= [5 037 690 ]
M

= 559 743.33

Remember that the standard deviation is the square root of the variance ( 559743.33) =
748.16
P'1P-
The quartile deviation formula is
.

Q1 = 1580

Q3 = 2930

So Q3 – Q1 = 2930-1580 = 1350

Dividing this by 2 gives 675

Question 9

Given the set of data below, draw the box and whiskers plot

4, 17, 7, 14, 18, 12, 3, 16, 10, 4, 4, 11

Solution

First order the data

3, 4, 4, 4, 7, 10, 11, 12, 14, 16, 17, 18

Find values for Q1, Q2, Q3 and Q4


-
Q1 = (4 + 4) = 4
.

-
Q2 = (10 + 11) = 10.5
.

-
Q3 = = (14 + 16) = 15
.

…then draw

Question 10

The pollution level in the city centre at 06:00 is 25 parts per million particles and it grows
linearly by 30 parts per million particles every hour. Determine the linear equation that
relates the pollution y with elapsed time x.
Tip: The best way to solve such an equation is to look out for key words such as:

Mathematical Descriptive words examples


representation
Addition (+) Sum; Increased; And; Grows;
Added
Subtraction (-) Less than; Less
Multiplication (×) Times; Per
Equals to (=) Result

Solution

We are told that the variable y indicates pollution and the variable x indicates elapsed time.

The first line says “the pollution level in the city centre at 06:00 is 25 parts per million”

So from this alone we can say:

𝒚 = 𝟐𝟓

Then it continues to say “it grows linearly by 30 parts per million every hour”

From this statement, we know we need to add (+) something to our first equation. We also
know that we need to multiply (x) the number 30 with something [grows = plus; per =
multiply].

Notice that the last part is talking about elapsed time x (“every hour”). Which gives you what
you must multiply 30 with.

So, from this we finally get:

𝒚 = 𝟐𝟓 + 𝟑𝟎𝒙

Question 11

A tutor at a college is required to do teaching and research for at most 40 hours per week. His
research work pays R30 per hour and teaching pays R70 per hour. The tutor needs at least
R2000 per week for his own studies.

Let x be the number of hours he spent for research per week.

Let y be the number of hours he spent for teaching per week.

Derive the system of inequalities that shows the number of hours he can spend on research
and teaching.

Tip: The best way to derive a system of linear inequalities from word statements is to
remember the following:
Descriptive words Inequality
At least
Minimum ≥
Not smaller than
At most
Maximum ≤
Not more than
Greater than
More than >
Above
Less than
Smaller than <
Below

Solution

They ask us here to determine a “system” of inequalities. Meaning we have to derive more
than one inequality.

The first sentence tell us that the tutor is “required to do teaching and research for at most 40
hours per week”

From this statement we already know that x represents teaching hours per week and y
represent research hours per week. Remember that the descriptive word “and” means addition
(+).
We also know that “at most” is represented by the inequality “≤”.

So from this we get,

𝒙 + 𝒚 ≤ 𝟒𝟎

The second sentence tells us that his research pays 30 per hour and teaching pays R70 per
hour. It also tells us that he needs at least R2000 per week for his own studies.

Remember that the descriptive word “per” means multiplication (x).


For research we have 𝟑𝟎×𝒙 𝒉𝒐𝒖𝒓𝒔 𝒐𝒇 𝒓𝒆𝒔𝒆𝒂𝒓𝒄𝒉 𝒑𝒆𝒓 𝒘𝒆𝒆𝒌 = 𝟑𝟎𝒙
For teaching we have 𝟕𝟎 ×𝒚 𝒉𝒐𝒖𝒓𝒔 𝒐𝒇 𝒕𝒆𝒂𝒄𝒉𝒊𝒏𝒈 𝒑𝒆𝒓 𝒘𝒆𝒆𝒌 = 𝟕𝟎𝒚

And to complete our inequality, we take into account the last statement where “at least” is
represented by “≥”.

So from this we get,

𝟑𝟎𝒙 + 𝟕𝟎𝒚 ≥ 𝟐𝟎𝟎𝟎

Finally, our system of inequalities is:

𝒙 + 𝒚 ≤ 𝟒𝟎
𝟑𝟎𝒙 + 𝟕𝟎𝒚 ≥ 𝟐𝟎𝟎𝟎

Question 12

Consider the following diagram:

Derive an inequality that represents the above.

Tip:

An open circle indicates strictly less than “<” or strictly greater than “>”.
A closed or shaded circle indicates less than or equal to “≤” or greater than or equal to”≥”.

Solution

We are only dealing with x here meaning that the above diagram indicates that x lies between
two values.

Let’s consider the shaded circle. The line moves from the circle to the right, it includes
numbers that are greater than or equal to 4.
Let’s consider the open circle. The line moves from the circle towards the left, it includes
numbers that are less than 9.

Putting this together, we get:

𝟒≤ 𝒙 <𝟗

Question 13

Draw the graphical representation of the following inequalities


𝒙−𝒚>𝟐
𝒙 + 𝟐𝒚 ≥ −𝟐

Tip: There are few easy steps to follow when you need to draw a graph.
Step 1: Rewrite the inequality to make y the subject of the formula
Step 2: Find the y-intercept by making x = 0. This is the point (0; y)
Step 3: Find the x-intercept by making y = 0. This is the point (x; 0)
Step 4: Plot your points on the graph
Step 5: Colour or use lines to indicate the region of all (x; y) points that satisfy the inequality
Then the overlapping part of both inequalities is the solution.
Solution

Let’s follow the steps. Starting with rewriting the inequalities to obtain y on the left-hand
side:
For the first inequality it is
𝒙−𝒚>𝟐
−𝒚 > 𝟐 − 𝒙
𝒚 < −𝟐 + 𝒙
The sign changed from > to < because we divided by a negative number.
For the second inequality it is
𝒙 + 𝟐𝒚 ≥ −𝟐
𝟐𝒚 ≥ −𝟐 − 𝒙
𝟏
𝒚 ≥ −𝟏 − 𝒙
𝟐

The y- intercepts are:


Inequality 1: 𝒚 < −𝟐 + 𝒙 → 𝒚 < −𝟐 + 𝟎 → 𝒚 < −𝟐
(𝟎; −𝟐)
𝟏 𝟏
Inequality 2: 𝒚 ≥ −𝟏 − 𝒙 → 𝒚 ≥ −𝟏 − 𝟎 → 𝒚 ≥ −𝟏
𝟐 𝟐
𝟎; −𝟏

The x –intercepts are:


Inequality 1: 𝒚 < −𝟐 + 𝒙 → 𝟎 < −𝟐 + 𝒙 → 𝒙 > 𝟐
(𝟐; 𝟎)
𝟏 𝟏
Inequality 2: 𝒚 ≥ −𝟏 − 𝒙 → 𝟎 ≥ −𝟏 − 𝒙 → 𝒙 ≥ −𝟐
𝟐 𝟐
−𝟐; 𝟎

Now to graph the inequalities


Question 14
The quadratic function

𝒚−𝟔=𝟑 𝒙−𝟐 𝒙−𝟏

Can be rewritten in the form 𝑦 = 𝑎𝑥 . + 𝑏𝑥 + 𝑐.


Identify the constants a, b and c.

Tip: After multiplying out your equation, “a” will be the number before 𝑥 . , “b” will be the
number before x and “c” will be the stand alone number. Remember to also include the sign
before the number (positive or negative).

Solution

𝒚−𝟔=𝟑 𝒙−𝟐 𝒙−𝟏


𝒚 − 𝟔 = 𝟑 𝒙𝟐 − 𝒙 − 𝟐𝒙 + 𝟐
𝒚 − 𝟔 = 𝟑 𝒙𝟐 − 𝟑𝒙 + 𝟐
𝒚 − 𝟔 = 𝟑𝒙𝟐 − 𝟗𝒙 + 𝟔
𝒚 = 𝟑𝒙𝟐 − 𝟗𝒙 + 𝟏𝟐

From this we see that:


𝒂 = +𝟑 = 𝟑
𝒃 = −𝟗
𝒄 = +𝟏𝟐 = 𝟏𝟐

Question 15

Find the vertex of the following function

𝒚 = 𝒙𝟐 + 𝟏𝟒𝒙 + 𝟒𝟎

Tip: The vertex is also known as the turning point. The value of x at the vertex is calculated
1𝒃
by using the formula 𝒙𝒎 = .
𝟐𝒂
The value of y at the vertex is found by substituting the value of x into the quadratic function.

Solution

𝒚 = 𝒙𝟐 + 𝟏𝟒𝒙 + 𝟒𝟎

From our quadratic function, we already know that,


a=1
b = 14
c = 40
Therefore,
1𝒃 1𝟏𝟒 1𝟏𝟒
𝒙𝒎 = = = = −𝟕
𝟐𝒂 𝟐×𝟏 𝟐

Substituting this back into the quadratic function,

𝒚 = 𝒙𝟐 + 𝟏𝟒𝒙 + 𝟒𝟎
𝟐
𝒚 = −𝟕 + 𝟏𝟒 −𝟕 + 𝟒𝟎

𝒚 = 𝟒𝟗 − 𝟗𝟖 + 𝟒𝟎 = −𝟗

Finally, the vertex is at the point (−𝟕; −𝟗)

Question 16

Simplify the expression

𝒍𝒐𝒈𝟕 𝟒𝟗1𝟏

Tip: A log is just another way to write an exponent! We can use three easy steps to solve
such problems.

Step 1: Set the log equal to x


Step 2: Use the definition of a log to write the equation in exponential form
i.e. 𝒊𝒇 𝒚 = 𝒂𝒙 𝒘𝒊𝒕𝒉 𝒂 > 𝟎 𝒂𝒏𝒅 𝒂 ≠ 𝟏, 𝒕𝒉𝒆𝒏 𝒍𝒐𝒈𝒂 𝒚 = 𝒙
Step 3: Solve for x

Solution

𝟏
First we notice that any expression written as 𝒂1𝟏 , can also be written as
𝒂
𝟏
So, 𝒍𝒐𝒈𝟕 𝟒𝟗1𝟏 = 𝒍𝒐𝒈𝟕
𝟒𝟗

Now we follow the steps.

Let’s set our log to x,

𝟏
𝒍𝒐𝒈𝟕 =𝒙
𝟒𝟗
Then use the definition to write it as an exponent,

𝟏
𝟕𝒙 =
𝟒𝟗
And now we solve for x,
The best way to solve for x is to think about how the equation on the right hand side can be
written in a similar form to the left hand side. For example, we know that:

𝟒𝟗 = 𝟕𝟐

So,

𝟏 𝟏
𝟕𝒙 = = 𝟐
𝟒𝟗 𝟕
Which already know can be written as:

𝟕𝒙 = 𝟕1𝟐

And finally,

𝒙 = −𝟐

Question 17

Find the equation of the line that passes through the points (-3; 8) and (4,-2)

Tip: The basic form a linear equation is 𝒚 = 𝒂𝒙 + 𝒃

Where a is the slope of the line and b is the intercept on the y – axis.
𝒚 1𝒚
To find the slope a line, we use the formula 𝒂 = 𝟐 𝟏
𝒙𝟐 1𝒙𝟏
To find b, since the line passes through both given points, either point can be used to find b
by substituting the x and y value of the selected point into our line after having found a.

Solution

𝑳𝒆𝒕 𝒙𝟏 ; 𝒚𝟏 = −𝟑; 𝟖 𝒂𝒏𝒅 𝒙𝟐 ; 𝒚𝟐 = (𝟒; −𝟐)

Let’s start with finding a,


−𝟐 − 𝟖
𝒂=
𝟒 − −𝟑
−𝟏𝟎
𝒂=
𝟕
This reduces our general expression to:
𝟏𝟎
𝒚=− 𝒙+𝒃
𝟕

To find b, let us use the point 𝒙𝟐 ; 𝒚𝟐 = (𝟒; −𝟐), remember you can use either point and
you will still get to the same answer. So,
𝟏𝟎
𝒚=− 𝒙+𝒃
𝟕
𝟏𝟎
𝒚𝟐 = − 𝒙 +𝒃
𝟕 𝟐
𝟏𝟎
−𝟐 = − (𝟒) + 𝒃
𝟕
𝟏𝟎×𝟒
−𝟐 = − +𝒃
𝟕
𝟒𝟎
−𝟐 = − +𝒃
𝟕
𝟐𝟔
=𝒃
𝟕

The expression for the line passing through the points (-3; 8) and (4;-2) is therefore

𝟏𝟎 𝟐𝟔
𝒚=− 𝒙+
𝟕 𝟕

Question 18

It is assumed that consumption C depends on income I and this relationship takes the form of
the linear function
𝑪 = 𝒂𝑰 + 𝒃
When I is R500, C is R550. When I is R1000, C is R800. Find the linear function that is
parallel to the above.

Tip: It makes it easier to solve such equations if we consider the characteristics of a straight
line

Specific cases of a straight line Characteristics and descriptions


Two lines that are parallel Both lines will have the same slope
A straight line that passes through There is no constant term (b-value) in
the origin the equation
A line that is parallel to the x- Expression is y = b where b is the
axis. intercept on the y-axis
Two lines that intersect at One line has a positive slope while the
the origin other has a negative slope – both with
b= 0
A line parallel to the y-axis. Expression is x = c where c is the
intercept on the x-axis.

Solution

We know that when I = 500, C = 550 and when I =1000, C = 800. Giving us the following
points
𝑰𝟏 ; 𝑪𝟏 = (𝟓𝟎𝟎; 𝟓𝟓𝟎)
𝑰𝟐 ; 𝑪𝟐 = (𝟏𝟎𝟎𝟎; 𝟖𝟎𝟎)
We know that any line parallel to our equation will have the same slope, so let us find the
slope a.
𝑪𝟐 1𝑪𝟏 𝟖𝟎𝟎1𝟓𝟓𝟎 𝟐𝟓𝟎 𝟏
𝒂= = = =
𝑰𝟐 1𝑰𝟏 𝟏𝟎𝟎𝟎1𝟓𝟎𝟎 𝟓𝟎𝟎 𝟐

So finally, our linear function will be parallel to any other linear equation of the form
𝟏
𝑪= 𝑰+𝒃
𝟐

Question 19

Consider the following table


Item 2010 2012
Price Quantity Price Quantity
A 260 200 520 380
B 250 650 265 350

Calculate the Laspeyres price index for 2012 using 2010 as the base year.
Tip:
We know that the Laspeyres price index is calculated as

𝒑𝒏 𝒒𝟎
𝑷𝒍 𝒏 = ×𝟏𝟎𝟎
𝒑𝟎 𝒒𝟎
Where,
𝒑𝒏 𝑖𝑠 𝑡ℎ𝑒 𝑝𝑟𝑖𝑐𝑒 𝑖𝑛 𝑡ℎ𝑒 𝑛𝑡ℎ 𝑦𝑒𝑎𝑟
𝒑𝟎 𝑖𝑠 𝑡ℎ𝑒 𝑝𝑟𝑖𝑐𝑒 𝑖𝑛 𝑡ℎ𝑒 𝑏𝑎𝑠𝑒 𝑦𝑒𝑎𝑟
𝒒𝟎 𝑖𝑠 𝑡ℎ𝑒 𝑞𝑢𝑎𝑛𝑡𝑖𝑡𝑦 𝑖𝑛 𝑡ℎ𝑒 𝑏𝑎𝑠𝑒 𝑦𝑒𝑎𝑟

Solution
It is easier to answer this question by constructing a table
Item 2010 2012
𝒑𝟎 𝒒𝟎 𝒑𝒏 𝒒𝒏 𝒑𝟎 ×𝒒𝟎 𝒑𝒏 ×𝒒𝟎
A 260 200 520 380 260 x 200 = 520 x 200 = 104000
52000
B 250 650 265 350 250 x 650 = 265 x 650 = 172250
162500
Total 𝒑 𝒒 𝒑𝒏 𝒒𝟎 = 𝟐𝟕𝟔𝟐𝟓𝟎
𝟎 𝟎
= 𝟐𝟏𝟒𝟓𝟎𝟎

Therefore, our Laspeyres price index for 2012 is calculated as

𝒑𝟐𝟎𝟏𝟐 𝒒𝟐𝟎𝟏𝟎 𝟐𝟕𝟔𝟐𝟓𝟎


𝑷𝒍 𝟐𝟎𝟏𝟐 = ×𝟏𝟎𝟎 = ×𝟏𝟎𝟎 = 𝟏𝟐𝟖. 𝟕𝟗
𝒑𝟐𝟎𝟏𝟎 𝒒𝟐𝟎𝟏𝟎 𝟐𝟏𝟒𝟓𝟎𝟎
Question 20

At a certain point in time the South African rand (R) is converted to the American dollar ($)
at a rate of
𝟏. 𝟎𝟎$ = 𝑹𝟔. 𝟓𝟎
If an article costs $15 in the USA, what is the number of articles that can be bought for
R2 535?

Tip: To answer such questions, always convert the total amount you have to the currency of
the goods. The goods in this case are “articles” which cost $15, so we must convert our total
rands of R2 535 to dollars so that we can answer the question.

Solution

Let us first convert our total of R2 535 to dollars.

To do so, the first thing we must do is determine the exchange rate in dollars.

This can be interpreted as a ratio:

𝒓𝒂𝒏𝒅 ∶ 𝒅𝒐𝒍𝒍𝒂𝒓
𝟔. 𝟓𝟎 ∶ 𝟏
𝟔. 𝟓𝟎 𝟏

𝟔. 𝟓𝟎 𝟔. 𝟓𝟎
𝟏
𝟏∶
𝟔. 𝟓𝟎

𝟏
Hence 1 rand is equal to dollars. This is our exchange rate.
𝟔.𝟓𝟎
If we multiply both sides of our ratio by 2 535 we get,

𝟏
𝟏×𝟐𝟓𝟑𝟓 ∶ ×𝟐𝟓𝟑𝟓
𝟔. 𝟓𝟎
𝟐𝟓𝟑𝟓 ∶ 𝟑𝟗𝟎

This means out of 2535 rands we have 390 dollars to spend on articles.
We already know that one article costs $15 and now we also know that we have $390 to
spend.

Therefore, the number of articles we can afford is equal to:

𝟑𝟗𝟎
= 𝟐𝟔 𝒂𝒓𝒕𝒊𝒄𝒍𝒆𝒔
𝟏𝟓
Question 21

You go into a foreign exchange bureau to buy US dollars for your holiday. You exchange
£200 and receive $343. When you get home you discover that you have lost the receipt. You
know that the exchange bureau charges a fixed £4 fee on all transactions. Calculate the
exchange rate used for your money.

Tip: The easiest way to tackle this kind of exchange rate question is to use a ratio.

Solution

£:$
We exchanged £200 but the question tells us that the bureau charges us £4 for a transaction.
So in total we gave the bureau £200 + £4 = £204
Therefore,
£𝟐𝟎𝟎 + £𝟒 ∶ $𝟑𝟒𝟑

£𝟐𝟎𝟒 ∶ $𝟑𝟒𝟑

£𝟐𝟎𝟒 $𝟑𝟒𝟑

𝟐𝟎𝟒 𝟐𝟎𝟒

£𝟏 ∶ $𝟏. 𝟔𝟖𝟏

The exchange rate is therefore:


£𝟏 = $𝟏. 𝟔𝟖𝟏

Question 22

Differentiate the function

Ž• 2
f(x) = − + 𝑥' − − 3x + 5
. 0

2
Tip: Transform to 4x-1 before differentiating.
0

Solution
Ž•
The function f(x) can first be expressed as f(x) = − + 𝑥 ' – 4x-1− 3x + 5
.

2
Since = 4x-1
0

Differentiating gives
Ž‘
f1(x) = -6 + 3𝑥 . – (-1)4x-2 – 3
.

Hence
2
f1(x) = -3x5 + 3x2 + -3
0D

Question 23

Suppose that the cost in rand for a daily production of x kilograms of copper is given by the
function
-
C(x) = - x2 + 150x + 2200.
.)

Calculate the marginal cost to produce the 1001th kilogram of copper.

Tip: you have to substitute 1000kg into the marginal cost function in order to determine the
marginal cost for the 1001th kg produced.

Solution

Differentiating the cost function


-
C(x) = - x2 + 150x + 2200
.)

Gives the marginal cost function


-
C1(x) = - (2) x + 150
.)
-
C1(x) = - x + 150
-)

Substituting 1000kg into the marginal cost function gives the marginal cost for the 1001th kg
produced as follows
-
C1(1000) = - (1000) + 150
-)

C1(1000) = 50
Hence the cost of producing the 1001th kilogram of copper is R50.

Question 24

The profit function to produce and sell x chairs is given by

p(x) = -x2 + 100x – 20

Calculate the number of chairs that have to be produced and sold in order to maximize the
profit.

Tip: Equate the marginal profit function to zero and solve for x.
Solution

Firstly calculate the marginal profit function by differentiating the profit function
p(x) = -x2 + 100x – 20 to obtain

p1(x) = -2x + 100

The number of units that must be produced and sold to maximize the profit is obtained by
equating the marginal profit to zero and solving for x.

Hence P1(x) = 0

-2x + 100 = 0 solving gives

2x = 100

x = 50

Question 25

Lindi invested R10 000 at 10% per annum simple interest for the first 5 years and thereafter
at 15% per annum simple interest for the next 3 years. Calculate the value of the investment
at the end of 8 years.

Tip: Use the formula S = P(1+RT) to find the accumulated sum after 5 years. This amount
will then be used as the principal for the next 3 years.

Solution

We will use the formula S = P (1+RT) to find the accumulated sum after 5 years. This
amount will then be used as the principal for the next 3 years.

P = R10 000

R = 10% = 0.10

T = 5 years

S = P (1+RT)

S = 10000(1+ 0.10(5))

S = 10000(1.5)

S = 15 000

This amount is used as the principal for the second part of the investment to find the new
accumulated amount as follows

P = R15 000
R = 15% = 0.15

T = 3 years

S = P (1+RT)

S = 15000(1+ 0.15(3))

S = 15000(1.45)

S = 21 750

Question 26

Peter receives R16 375 when taking out a loan from a lender with the simple discount rate of
14.5%. The loan has to be repaid in fifteen months’ time. Calculate the amount that he as to
repay.

Tip: Note that the time is given in months. You have to convert it to years by dividing by 12.
Secondly, you can use the discount formula P = S (1 - dT) to solve for S.

Solution

In this question, we have to convert the time to years.

P = R16 375

d = 14.5% = 0.145
-3
T= = 1.25 years
-.

P = S (1 - dT)

Substituting into the formula gives

16 375 = S (1 – 0.145(1.25))

16 375 = S (1 – 0.18125)

16 375 = S (0.81875)
-= '<3
S=
).;-;<3

S = 20 000

Question 27

You invest R12 000 at 15% simple interest per year. The investment amounts to R20 100.
Calculate the period of the investment in months.

Tip: Use the future value formula S = P (1 + RT) to find the period of investment, T.
Solution

Use the future value formula S = P (1 + RT), to find the period of investment, T as follows

S = 20 100

P = 12 000

R = 15% = 0.15

S = P (1 + RT)

20 100 = 12 000 (1 + 0.15T) hence

20 100 = 12 000 + 1800T

20 100 – 12 000 = 1800T

8 100 = 1800T
; -))
T=
-;))

T = 4.5 years.

In order to get the time in months, we will multiply 4.5 by 12 to get 54 months.

Question 28

You invest R15 000 at 16.5% per year, compounded monthly. The investment amounts to
R40 097.17. Calculate the period of the investment in months.

Tip: Use the formula S = P (1 + R)T, to find the investment period. You will need to use
logarithms in order to find T.

Solution

Future value, S = 40 097.17

Present value, P = 15 000


-=.3% ).-=3
Effective Interest rate per month = = = 0.01375 (it is the effective interest rate
-. -.

paid every month since it is compounded monthly)

Substituting into the formula S = P (1 + R)T gives

40097.17 = 15 000(1 + 0.01375)T

40097.17 = 15 000(1.01375)T
2))M<.-<
= (1.01375)T
-3 )))

2.67314 = (1.01375)T

Taking logs of both sides in order to solve for T gives

Log (2.67314) = T Log (1.01375)


“”•..=<'-2
T=
–—˜ -.)-'<3

T = 72 months

Question 29

Johan will need R35 000 for his daughter’s first year’s university fees starting in 4 years’
time. How much money should he invest now, to provide for the first year of study if the
interest rate is calculated at 18% per year, compounded annually?

Tip: Use the formula S = P (1 + R)T to find the present value, P.

Solutions

Future value, S = 35 000

Time, T = 4 years

Effective Interest rate per year 18% = 0.18 (it is the effective interest rate paid every year
since it is compounded annually)

Substituting into the formula S = P (1 + R)T gives

35 000 = P (1 + 0.18)4

35 000 = P (1.18)4
'3 )))
P=
-.-;@

P = 18 052.61

We could have easily solved the question using the recommended calculator as follows

2ndF CA

2ndF P/Y 1 ENT

ON/C

+/- 35 000 FV

18 I/Y
4 2ndF xP/Y N

COMP PV

PV = 18 052.61

Question 30

Thabo starts a small business and takes out a loan of R50 000. He agrees to repay R20 000
at the end of 2 years, and then follow up with another payment of R25 000 one year later.
After 5 years he expands his business and takes out an additional loan of R75 000, and agrees
to repay R55 000 one year later and the rest of his debt at the end of the third year. If the
interest rate is 16% per year compounded annually, calculate the final payment.

Tip: The key thing in this question is to draw a timeline of this scenario. Secondly, find the
accumulated amounts of both the debts and payments at the time the last payment is made.
Finally, equate total payments to total obligations and solve for the unknown.

Solution

The future value of the payments at the end of 8 years must be equal to the future value of
both loans at the end of 8 years.

Hence, at time T = 8, the future values are calculated using S = P (1 + R)T as follows:

20 000 (1 + 0.16)6 + 25 000 (1 + 0.16)5 + 55 000 (1 + 0.16)2 + X = 50 000 (1 + 0.16)8 +


75 000 (1 + 0.16)3

20 000 (1.16)6 + 25 000 (1.16)5 + 55 000 (1.16)2 + X = 50 000 (1.16)8 + 75 000 (1.16)3

48 727.93 + 52 508.54 + 74008.00 + X = 163 920.74 + 117 067.20

175 244.47 + X = 280 987.94

X = 280 987.94 – 175 244.47

X = 105 743.47

Hence the final payment is R105 743.47

Question 31

John pays R2 500 at the end of each quarter into a sinking fund for 9 years. Interest is 18%
per year compounded quarterly. Calculate the amount John will receive at the end of 9 years.

Tip: use the future value annuity formula.


Solution

Payment, R = 2 500
-;% ).-;
Effective Interest rate per quarter = = = 0.045 (it is the effective interest rate paid
2 2
every quarter since it is compounded quarterly)

Number of payments, n = 36, (9 x 4 quarters in a year)

We will use the Future value annuity formula to find the future value, S as follows:
-?H ™ 1-
S=R
H

-?).)23 š• 1-
S = 2 500
).)23

S = 215 409.91

We could have easily solved the question using the recommended calculator as follows

2ndF CA

2ndF P/Y 4 ENT

ON/C

+/- 2 500 PMT

18 I/Y

9 2ndF xP/Y N

COMP FV

FV = 215 409.91

Question 32

Nandi wants to invest R3 500 at the end of each second quarter for 7 years. The investment
will earn 11.5% per year, compounded half yearly. Calculate how much she will accumulate
at the end of the 7 years.

Tip: use the future value annuity formula. Note that paying an amount at the end of each
second quarter is the same as paying it half yearly.

Solution

Payment, R = 3 500
--.3% ).--3
Effective Interest rate per half year = = = 0.0575 (it is the effective interest rate
. .
paid every half year since it is compounded half yearly)
Number of payments, n = 14, (7 x 2 half years in a year)

We will use the Future value annuity formula to find the future value, S as follows:
-?H ™ 1-
S=R
H

-?).)3<3 C@ 1-
S = 3 500
).)3<3

S = 72 275.62

We could have easily solved the question using the recommended calculator as follows

2ndF CA

2ndF P/Y 2 ENT

ON/C

+/- 3 500 PMT

11.5 I/Y

7 2ndF xP/Y N

COMP FV

FV = 72 275.62

Question 33

Anthony wants to invest the same amount of money at the end of each quarter for five years.
The investment will earn 9% interest per year, compounded quarterly. Calculate the amount
of each quarterly payment if he wants to accumulate R50 000 at the end of 5 years.

Tip: Use the future value annuity formula to calculate the quarterly payment.

Solution

Future value, FV = 50 000


M% ).)M
Effective Interest rate per quarter = = = 0.0225 (it is the effective interest rate paid
2 2
every quarter since it is compounded quarterly)

Number of payments, n = 20, (5 x 4 quarters in a year)

We will use the Future value annuity formula to find the payment, R as follows:
-?H ™ 1-
S=R
H

-?).)..3 D› 1-
50 000 = R
).)..3
50 000 = 24.9115R
3) )))
R=
.2.M--3

R = 2 007.11

Question 34

Andile takes out a five year loan to buy her dream car. The loan is financed at 15% per year
compounded monthly and her monthly payments are R4 150. In addition to the loan, she
makes a deposit of R30 000 on the car. Calculate the price of the car.

Tip: use the present value annuity formula to find the present value of the payments. Then
add R30 000 to get the price of the car.

Solution

Payment, PMT = 4 150


-3% ).-3
Effective Interest rate per month = = = 0.0125 (it is the effective interest rate paid
-. -.
every month since it is compounded monthly)

Number of payments, n = 60, (5 x 12 months in a year)

We will use the present value annuity formula to find the present value, P as follows:
-?H ™ 1-
P=R
H -?H ™

-?).)-.3 •› 1-
P = 4 150
).)-.3 -?).)-.3 •›

-.)-.3 •› 1-
P = 4 150
).)-.3 -.)-.3 •›

P = 174 443.56

Hence the cost of the car is R204 443.56 (174 443.56 + 30 000 deposits)

We could have easily solve the question using the recommended calculator as follows

2ndF CA

2ndF P/Y 12 ENT

ON/C

+/- 4 150 PMT

15 I/Y

5 2ndF xP/Y N
COMP PV

PV = 174 443.56

Questions 35, 36 and 37 are based on the following information

Sandra buys a house for R480 000. She has to pay a 20% deposit and takes out a loan from
the bank for the balance. The loan is amortised over a period of 20 years by means of equal
monthly payments. The interest rate is 21% per year, compounded monthly.

Question 35

Calculate Sandra’s monthly payment.

Tip: you have to subtract the deposit amount from R480 000 in order to know the value of
the loan. Then use the present value annuity formula to find the monthly payment.

Solution

Deposit amount = 20% of 480 000 = 96 000

The present value, PV = R384 000 (R480 000 – R96 000) since he paid a deposit.
.-% )..-
Effective Interest rate per month = = = 0.0175 (it is the effective interest rate paid
-. -.
every month since it is compounded monthly)

Number of payments, n = 240, (20 x 12 months in a year)

We will use the present value annuity formula to find the payment, R as follows:
-?H ™ 1-
P=R
H -?H ™

-?).)-<3 D@› 1-
384 000 = R
).)-<3 -?).)-<3 D@›

-.)-<3 D@› 1-
384 000 = R
).)-<3 -.)-<3 D@›

384 000 = 56.2543R


';2 )))
R=
3=..32'

R = 6 826.15

We could have easily solve the question using the recommended calculator as follows

2ndF CA

2ndF P/Y 12 ENT

ON/C
+/- 384 000 PV

21 I/Y

20 2ndF xP/Y N

COMP PMT

PMT = 6 826.15

Question 36

Considering an amortisation schedule, calculate the interest due at the end of the first two
months.

Tip: Draw up an amortisation table for two months and determine the interest due.

Solution

The amortization table for the loan is as follows

Month Outstanding balance at Interest due at the payment Principal repaid


beginning of month end of the month

1 384 000 6 720 6 826.15 106.15

2 383 893.85 6 718.14 6 826.15 108.01

Note that the interest due at the end of month one is

384 000(1.0175) – 384 000 = 6 720

The principal repaid = 6 826.15 – 6 720 = 106.15

Principal outstanding at the beginning of year two = 384 000 – 106.15 = 383 893.85

Interest due at the end of the 2nd month = 383 893.85(1.0175) – 383 893.85 = 6 718.14

The principal repaid = 6 826.15 – 6 718.14 = 108.01

Question 37

Calculate the outstanding principal after 10 years.

Tip: find the present value of the last ten years of monthly instalments that have not yet been
paid using the present value annuity formula.

Solution

The outstanding balance is calculated by working out the present value of the last 120 (240-
120) remaining monthly payments. From the previous calculation, we know that the monthly
payment is R6 826.15
We will use the present value annuity formula to find the present value, P as follows:
-?H ™ 1-
P=R
H -?H ™

-?H ™ 1-
P=R
H -?H ™

-?).)-<3 CD› 1-
P = 6 826.15
).)-<3 -?).)-<3 CD›

-.)-<3 CD› 1-
P = 6 826.15
).)-<3 -.)-<3 CD›

P = 6 826.15 x 50.0171
P = 341 424.08
We could have easily solve the question using the recommended calculator as follows
2ndF CA
2ndF P/Y 12 ENT
ON/C
+/- 384 000 PV
21 I/Y
20 2ndF xP/Y N
COMP PMT
PMT = 6 826.15
RCL N
-120 = N
COMP PV
PV = 341 424.08
May-June 2018 P2 QMI1500
memo

Question Answer
1 2
2 3
3 2
4 3
5 4
6 2
7 1
8 2
9 4
10 4
11 3
12 4
13 3
14 2
15 1
16 2
17 3
18 3
19 4
20 3
21 2
22 3
23 1
24 2
25 3
26 1
27 4
28 4
29 3
30 1
QUESTION 1

5 9 3 5
+ ÷ X OPTION ( 1)
9 5 4 2

5 9 4 5
9
+5 X3 X2

5 3
9
+5 X2X5

5 3
9
+ 5 X 10

5
9
+6

𝟓
6𝟗

QUESTION 2

OPTION (2 )

9 − 𝑋2

𝑋2

1
3 − 𝑋2 (2)
1
𝑋2 (2)

3−𝑋
𝑋

3
𝑋
-1

1
QUESTION 3

7 Py = 210
Since order is important we use permutations

8000

2 1 0 1 2 5 8 10

3months ago nov 10500

S = p (1+ rt)
13
S = 8000 (1 + 0.115 x )
12

S = 8996.67

Lunwabo is still short with 1503.33 (10500 – 8996.67) (OPTION (4)

QUESTION 5

20000

3 6 9 12 15 18 21 24 27 30 33 36 MONTHS

NOW

2
P = S (1 – DT)
27
20 000 (1 – 0.125 X 12

= 14 375 OPTION (2)

QUESTION 6
𝑃𝑛
1= 𝑝𝑜
x 100

𝑥
7000
x 100 = 120

X x 100 = 120 x 7000

100x = 840 000

X = 8400

9000 – 8400 = 600 increase OPTION (5)

QUESTION 7

Fv = 219 464.29
9.25
 = 12

N = 10 x 12

Pmt = 1118.16 OPTION (1)

3
QUESTION 8 2

QUESTION 9

QP = 𝑄3−𝑄1
2
Q1 = ¼ (23 + 1 )

= 6TH = 10

2 5 7 8 9 10 10 12 12 14 15 15 16 18 20 23 25 28 28 30 30 33 35

QUESTION 3

= 2 ¾ ( 23 + 1 ) = 18TH OBSERVATION = 28
28−10
Qn = 2
=9

ANS (2 )

QUESTION 10

STEM LEAF
0 25789
1 002245568
2 03588
3 0035

Ans (2)

QUESTION 11

35

28

15

10

4
QUESTION 12 (4)

OPTION (4)

QUESTION 13

(a) – 2 ≤ x , 2
(b) X > 0
(c) X <0 option (4)

QUESTION 14

15 000 x 70 % = 10 5000

Dank =3

Friend = 5

Doctor = 7

15 option (1)

7
Doctor = 15 x 10 500 = 4900

QUESTION 15

He will save
5
Friend X 10 500 = 3500
15

3500 X 12 % = 420 option ( 4)

5
QUESTION 16

3
4X + √196 ≤ 7X - √343

= 4X + 14 ≤ 7X – 7

= 14 + 7 ≤ 7X – 7

= 21 ≤ 3X

=7≤X

=X≥7 OPTION (4)

QUESTION 17
800 500 300
- = = - 60
10 15 −5

D = -60P + 1400 OPTION (3)

QUESTION 18
The vertex is at x = 0 OPTION (3)

QUESTION 19
−𝑏
Vertex = 2𝑎

−1 −1 1
= 2 (−2) = −4 = 4
= 0.25 OPTION (4)

QUESTION 2 0
C (X) = 0.78X + 150 OPTION (1)

6
QUESTION 21
278.45 = 0.78X + 150
278.45 – 150 = 0.78X

128.45 0.78𝑋
0.78
= 0.78

X = 164.68 OPTION (1)

QUESTION 22
0.78 (80) = 62.40 OPTION (3)

QUESTION 23
X=0
Y = -3 (0)2 + 2 (0) + 5
Y=5
(0.5) OPTION (1)

QUESTION 24
−10
Y = -3X2 + 2X + 5 −6

−𝑏√𝑏2 −4𝑎𝑐 𝟐
- 1
2𝑎 𝟑

−2±√22 −4(−3)5
2 (−3)
x1 = -1 y1 = 0

−2±√4+60 2
−6
x2 = 1 3 y 2 = 0 option (4)

−2 ± 8
−6
−2 − 8
−6

7
Question 25

3 OPTION (2)

-3 -2 -1 0 1 2 3 4 5

QUESTION 26

215 + 300 + 250 + 150 + 300 + 220 + 180 + 200 + 170 + 190 + 260 + 350 + 100 + 400 + 80 = 3365

3365 224.33 𝑀𝐸𝐴𝑁


= OPTION (3)
15

8
QUESTION 27

Mode is 300 option (1)

QUESTION 28
1
(N + 1 )
2

= ½ (15 +1 ) = 8TH Number

80, 100, 150, 170, 180, 190, 200, 215, 220, 250, 260, 300, 300, 350, 400

Medium = 215

QUESTION 29

Range = Highest minus lowest

= 400 – 80

= 320

QUESTION 30

49C6 = 13 983 816 OPTION (2) Using calculator

QUESTION 31

13983 816 X 2.50 = 34 959 540 – 1000 000

= 33 959 540 108 OPTION (3)

QUESTION 32

26.99 X 7.8841 = 212.79 OPTION (4)

QUESTION 33

X + 2Y < 40 (1)

9
10X + 25Y ≥ 250 (2)

X ≥ 0

Y ≥ 0 OPTION (2)

QUESTION 34

500 930

0 2 3 4 5 6 7 8

Q35 MONTHS

FV = 500
20
I = 12

N=1

PV = 491.80 – 500 = 8.20

Fv = 930
20
I = 12

N=3

Pv = 885.01 – 930 = 44.99

Interest saved = 8.20 + 44.99 = 53.19 option (3)

10
QUESTION 35

Pv = 500
20
I = 12

n=1

fv = 508.33

fv = 930
20
I=
12

N=1

Pv = 914.75

He will have to pay 1423.08 (914.75 +508.33) option (3)

QUESTION 36

P(X) = 20X2 + 8000X + 50

= 40X2-1 + 8000X 1-1

X = -40X + 8000 OPTION

11
QUESTION 37

400 = -140X + 8000

40X = 8000 – 400

40𝑋 7600
=
40 40

X = 190 OPTION

QUESTION 38
𝑋4
F(X) = + 5X-2 + 15
6

= -4X4-1 + 5 (-2) X -2-1 + 15


−4𝑋3
= – 10X-3
6

= -2 (𝟐 2
X + 5X -3 ) OPTION (3)
𝟑

QUESTION 39

PV = 65 000
15
I = 12

N = 3x12

Pmt = 2253.25 OPTION (4)

12
QUESTION 40

Interest due end of 4th Month.

Balance end of month 3 = 60 623.51


1
60 623.51 x 15% x 12

= 757.79 option (2)

13
QMI1500 MAY / JUNE 2013-10-24

1. 8X + 8 3X

23X X 4 (X+2)

= 8 (X+3X) (5)

23XX 4X 4

= 84X

8(4X) 4

3
2. ( I ) ¾ ( √1278 - 0.125 + 9.9342)

= ( 1278 )1/3 – 2.48355


= 6.2763

( II) 0.475 KM

𝟒𝟖
(III) (5)
𝟏𝟎𝟎

10! 10! 10𝑋9𝑋8𝑋7!


3. 10 P3 = = =
(10−3)! 7! 7!

= 720 (4)

4! 3! 2!
4. 4 C2= X 3C1 = X 2C1 =
2;2; 2! 1!!

4+3+2! 4𝑥33
= =6
2!2! 2!

3𝑋2! 3
= =3
2! 1

1
2𝑥1 2
= =2
1! 1
𝟔 𝑿 𝟑𝑿 𝟐 = 𝟑𝟔 (5)

4
5. ∑ (-1) M+1(M2 + 2 m)
= [ ( -1 ) 1+1 ( 12 + 2 ( 1 ) ] + [ (-1) 2+1 (22+ 2 (2) ]
+ [ (-1) 3+1 (32 + 2 (3))] + [ ( -1)4+1(42+2(4) ]
= [ (1.3] + [-1.8] + [1.15] + [ -1 .24]
= 3-8+15 -24
= -14 (2)

6. (3)

7. ( 4 )

8. Proof a = 1 b = 4 (1.10)
D (+) = at2 + bt +5
= t2 + 4t + 5
=12 + 4(1) +5
= 10

9. (5)

10.Albentina
Range is 25
Maximum mark 78 (78-25) = 53

Frananah
3 (7+1)
Q3 =
4

= 6th observation (1 )

Med. 68

2
Q3 = (68 + 9) = 77

𝑏
11.
−2𝑎

7
2 (4)
𝟕
=
𝟖

4
Y = 4 (7/8 )2 – 7 ( ) + 3
8

49 28
= 4( )- +3
64 8

49 7
= - +3
16 2

𝟒𝟏
= (5)
𝟏𝟔

(12) Y = ax

13.) ( 1)
2x + y = -9
-2 + 2y = -15
3𝑦 24
=-
3 3

Y=-8 (3)

2x + (-8) = - 9
2 x – 8 = -9
2𝑥 −1
=
2 𝑥
𝟏
X=-
𝟐

3
14. ( 2)

15. R420

R35

R385 Mat

16. SDP (3)

17. PROOF 5X + 3Y – 15 ≤ 0
5 (0 ) + 3 (0) – 15 ≤ 0
– 15 ≤ 0 (2)
The equation satisfy the rules.

18. Interest = R (186 000 - 150 000) 10 = PRT


= r36 000
36 000 = 150 000 x 0.12 x T

𝟑𝟔𝟎𝟎𝟎 𝟏𝟖𝟎𝟎𝟎 𝑿 𝑻
= ( 2)
𝟏𝟖𝟎𝟎𝟎 𝟏𝟖𝟎𝟎𝟎

𝑇 = 2 𝑌𝐸𝐴𝑅𝑆 = 24 𝑀𝑂𝑁𝑇𝐻𝑆

19. (1)
Costs 150000
Discounts (0.05 x 150000) 7500
142500
14 19950
VAT ( X 142500)
100
162450
1500
163950

4
20. P = R [ (1 + i)n – 1 ]

I ( 1 + I )n

0112
97000 = R [ ( I + ) (5X12) – 1)
12

0172 0112
(1+ ) (5X12) ]
12 12

R = 2157.71

21. S = P (1 + C )n
0.059
= 12000 (1 + )5
12

12297.91

22. S = ( p ( 1 + i) n
0.058 2
= 12650 (1 + )
1
= 14159. 95 (3)

( 1+𝑖 )𝑛−1
23. P=R [ ]
𝑖 ( 1+𝑖 )𝑛

01150 (3X12)
50000 = R [ 1 +( ) –1
12

0115 0115
(1+ ) (3X12) ]
12 12

50000 28.8472𝑅
=
28.8472 28.8472

R = 1733.266 (4)

5
24. [ 1733.66 X 36 – 50000 ]
= 12397159 (3)

25. (1 )

26. 6X + 16 ≤ 8 + 8X

6X – 8 X ≤ 8 - 16

−2𝑋 −8

−2 −2

X≤ 4 (5)

27.

P10 Q10 P12 Q12 P10XQ10 P10XQ12 P12 XQ10 P12 X Q12
A 260 200 520 380 52000 98800 104000 197600
B 250 650 265 350 162500 87500 172250 92750
214500 186300 276250 290350

𝟏𝟖𝟔𝟑𝟎𝟎
Laspeyres x 100 = 86.85 (3)
𝟐𝟏𝟒𝟓𝟎𝟎

290 350
28 Pausche = x 100 = 105.10 (2)
276250

29. ) (25000 x 1.6915)


= 42287.5 MXN (4)

30. (45+ 1125) = 1170

45 1 1125 25
= =
1170 26 1170 26

= 1;25 (4 )

6
31. Rat. 3: 5
5
x 1125
8
= 𝟕𝟎𝟑 𝒄𝒂𝒓𝒔 ( 3)

7 5
32. ( x 900 ) cm - ( x 900)
20 20

=90cm
=0.9m (4)

7 5 8
33. Volume = x 9000 x ( x 900 ) x ( x 900 )
20 20 20
= 2551500 cm3 (5)
= 25.51500 m3 (5)

34. F (x) = -3x 1 + 5x2 – 5x + 1


𝑑
(x) = - 3 (-1) x-1-1 + 5 (2) x 2- 1 - 5 x -1-1
𝑑𝑥
= 3𝑥 -2 + 10x -5
𝟑
= + 10x – 5 (2)
𝒙𝟐

1080+2000+1580+1540+2500+1800+1580
35. 𝑥
7

= 1725.71 (5 )

36. Mode = 1580 (3)

7
√(𝑥−𝑥)2
37. S =
𝑛−1

√(1080−11725.71 )2+(2000−1725.71)2…….
=
7−1

1174172
6
= 442.37 (5)

𝑄3−𝑄1
38. Quantile dev. =
2
1080,1540,1580, 1580, 1800, 2000, 2500
1
Q1= (7+1)
4
= 2ND OBSERVATION = 1540

39. ( 4) pg 180

40. (2)

8
QMI1500 MAY/ JUNE 2012

160
1. (2) 120
= 1.33

2. (32X30) M / 4M2
960𝑚2
= 4𝑚2
= 240 (1 )

1 3 1 1
3.(1 4 + 4 + 2 + 3 ) ÷ 2

17 2
= ÷
6 1

17 1
= 6
x2

17
= 12

𝟓
=1 (1)
𝟏𝟐

4. 56 + 5-2

75 + 710

= 56 + -2

75+10
𝟓𝟒
= 𝟕𝟏𝟓 (3)

25! 25! 25𝑥24𝑥23!


5. = = 23!
(45−2 )! 23!

= 575 (4)

1
6. ( 180 x 0.75) + (400 x 0.7)

= 550 (5)

7. 4x3 – 4x 2 + 10

= 4 (-2)3 – 4 (-2)2 + 10

= 4 (-8) – 4 (4) +10

= -32 – 16 + 10

= - 48+ 10

= - 38 (4)

8. x > 100

2𝑥 5 𝑥
9. 3
-2 =4

2𝑥 𝑥 5
3
-4 =2

5𝑥 5
12
=2

10𝑥 60
10
= 10

X=6 (3)

10. ( 3)

11. – 4 > -n

N<4 ( 1)

2
12. – 3a + b = -7

-3a -2b + -13


3𝑏 6
3
=3

B=2

-3 a + 2 = -7

- 3a = - 7 – 2
−3𝑎 −9
−3
= −3

A=3 (2)

13. 2x + y ≥ -3

2(0) + (0) -3 ≥ 0 . it does not satisfy the equation (4)

14. –x-2y-4<0

-0-2(0) -4<0

-4 <0 it satisfies the equation (2)

−3−6
15. 4− −2

−9
= 6

−𝟑
=
𝟐

−3
Y= 2𝑥
+c

3
-3 = -2 (4) + c

-3=-6+c

-3+6=c

3=c (1 )

3
16. 2x + 5y = 10

5y = - 2x + 10
2 10
Y = -5𝑥 + 5

𝟐
Slope = - (1)
𝟓

17. (3)

18.

-4a 1c

(2)

19. y = -11x2 + 350x

Y = -11 (9)2 + 350 (9)

= - 891 + 3150

=2259 (1 )

4
20. y = 5x2 – 12x 4

X=

+20 ± √12 2 − 4 (5)(4)


2 (5)

12± √64
X = 10

𝟏𝟐±𝟖
X= (3)
𝟏𝟎

21. 1 = prt

589 = p (0.095) (2)

589 0.19𝑝
=
0.19 0.19

P = 3100 (1)

𝑏
22. = - 29

( −4)
X = - 2 (−1)

X=-2

Y = - (-2)2 – 4 (-2) +5

= - 4 + 8+5

Y=9 (4)

5
0.10 0.12 (215+12)
23. 9000 (( 1 + ) (4x215) – (1 + ))
4 12

= 9000 (1.0130 – 1.02510 ) (4)

𝟎.𝟎𝟗 6
24. s = 25000 ( 1 + )
𝟐

25. p = s (11 – dt)


9
1650 = s (1 – 0.14 ( 12 ))

1650
S= 3
( 1−0.44(
4 ))

S = 1843.575

S= 1843.58 (4)

(𝐻𝑖)𝑛−1
26. s = r [ ]
𝑖

30000 = r [ (1 + 0.085 )(4x5) – 1 ]


4
0.085
4

30000
R=
241602

R = 1219.407

R = 1219.41

6
27. p = r [ (1 + I )n - 1 ]

I (1 + I )n

= 5842 [ 1 + 01105 ) (15x12) - 1

12 ]
01105 01105
12
(1+ 12
) (12x15)

= 5842+9014650

= 528496.9865

= 528496.99 (3)

0.10
28. 55000 x 2

= 2750.00 (2 )

29. Qn 29 (2)

Answer: look @ formula

𝑝𝑖𝑖 𝑥 𝑞11
𝑝𝑖𝑖 𝑥 𝑞09

cpi
140000
30. 08 140000 137.1 13711
x 100 = 102115.24

𝟏𝟓𝟐𝟎𝟎𝟎
11 152000 139.5 𝟏𝟑𝟗,𝟓
x 100 = 108960.57 (4)

7
25 15
31. € 45 - [( 1.34 ) + ( 1.34 ) ]

= € 45 - € 29.85

= € 15.1492

€ 15.15 (1 )

32. 0.236 + 250

59 people (2)

33. (4) (100 – 61) 87.87

34. (980 – 1140)2 + (1050 – 1140 )2 + (630 – 1140 )2 + (1120 – 1140 )2 + (1540 – 1140 )2 + (1260 –
1140)2 + (1400 – 1140)2

√5643400
S= 7−1

𝑛+1
35. = 2

7𝑥1
=
2

= 4 th (2)

Q1 med Q3

36. 630 , 980, 1050, 1120, 1260, 1400, 1540


1 (7+1) 3 (7+1 )
Q1 = 4
Q3 = 4

= 2 Observation = 6TH observation (2)

𝒂+𝒃+𝒄
37. 𝟑
(2)

38. (3 )

8
5
39. y (x) = -7 + 5x - 2𝑥 2 - 3x4

𝑑 5
𝑑𝑥
(x) = 0+ 5 - 2 (2) x2-1 – 3 (4) x 4-1

Y1 (x) = 5 – 5 x – 12x3 (2)

40. c1 (x) = 25 – 0.10x

C (70) = 250 – 0.10 (70)

C ( 70) = 25 – 7

C ( 70 ) = 18 (1)

9
10
11
QM1 OCTOBER / NOVEMBER 2010
𝑋+8
1. 5−𝑌
(1)

48
2. (2) e,y100

3 15
3. =
𝑥 24

72 15𝑥
15
= 15
(3)

X = 4,8
𝟒
X=4𝟓

𝟑 14
4. √5
25

=𝟑 139

25

𝟏𝟑𝟗 1/3 3√129


=( ) = 3√251 (6)
𝟐𝟓

= 1.7715

= 1.772

80
5. 90X0.8 = (60+30)mihs X 100
= 72mihs (4)

1
2500 800 2700
6. R T = 6000 , F = 6000 , S = 6000 (6000- (2500 + 800) =

2700
Sipho x 2000 (1)
6000

= 900

7. 100% - (45+15)%
40%
R2875 X 014
= R1150 (3)

8! 8! 8𝑋7𝑋6𝑋51
8. P3 = =
(8−311) 5! 5!

= 336 (3)

9. Y = 4X -2
Y = 4(0) -2
Y = -2
Y = -2

Y = 4X-2
0 =4X-2

4𝑋 −2
-4 = 4

X=½ (2)

10. Y axis 2
2
Y = 3 (-3) + 2
Y = -2+2
Y=0 (3)

2
3 4

11.  x2 + ( )3
1 1

6-2

 (22 + 32) + (2+3+4)3


= 4+9+(9)3
= 13+729
=742 (5)

12. (x) = x (1000 – x)


= 1000 – x2 (2)

𝑑𝑦
13. = 1000x-x2
𝑑𝑥
= 1000 − 2𝑥
= 1000 − 2𝑥 = 400

−2𝑥 400−1000
=
−2 −2

X = 300 (3)

14. (4)

3
15. Y = 4x2 – 8x + 3

𝑏
X=-
29

8
X=
2 (4)

8
X=
8

X=1 (2)

16. A<0 (1)

4
17. 5x+2y = 13
X + 2y = 9

4𝑥 4
4
=4

X=1 (3)

𝑏 −√𝑏2 −4𝑎𝑐
18. X = - 29 ± 2𝑎

D = b2 – 4ac

D = 82 – 4 (-4) (-3)
D –64 -48
D = 16

−𝑏− √𝑑
X=
2𝑎

−8−√16
= 2(−4)

−8+√16
=
2(−4)

−12
=
−8

−4
=
−8

=1 1/2

5
−𝑏− √𝑑
X=
2𝑎

−8−√16
= 2(−4)

−4
=
−8

19.

2 5

5X + 2Y ≥ 10

X+Y≤ S (1)

20. -1<X≤ 5 (1)

6
−5 25
21. 39 <27

135 759
- < 75 (1)
75

4
- 1 5< Q

𝟒
Q > - 1𝟓

22. Slope (1)

6x + 7 > 8x -3

6x -8x > -7-3

−2𝑥 −10
.
−2 −2

X>5 (1)

23. 6x + 7 > 8x -3
6x -8x > -7-3

−𝑥 −10
.
−2 −2

x>5 (2)

24. 1 = PRT
3
1 = 5000X0.12 X 12

= 150 (4)

7
25. P = S (1-DT)
= 5000 (1-0.15+2)
= 3500 (1)

26. (2)
X-3Y< 5
0 – 3(0) -5 < 0
-5<0 (2)

27. S = P (1+R)1
0.125 (5+12)
S= 12000 (1+ )
12
= 22346.59 (3)

INTEREST = 22346.59 – 12000


= 10346.59 (3)

28. S = P1 +R)R
0.12 3
= 2000 (1 + 12
)
2060.60 (1)

2000
29.P = 0.12 8
1+
12 )

= 1846197 (4)

30.Calculator . Financial

P=r [ (1+
0+4)𝑛−1
𝑛
𝑖 (1+𝑖)𝑛
]

= 500 [ (1+
0111
12
0111) (3𝑥12)−1

0111
12
(11 12 ) (3𝑥13)
]

8
15
31.Cv (b) = = 0.2676 = 26.78% (1)
56

32.Quantile duration : test 2


𝑄3−𝑄1 75−40 35
Q0 = = = = 17.5 (2)
2 2 2

33.RANGE 87 -10
=77 (4)

34. (2)

35.F (X) = 2X ½ + 5
𝐷
(2X1/2 ) = 2 (-1/3)X1/3-1
𝐷𝑋
2 -4/3
= −
3𝑋

𝑫
(5)= 0 (2)
𝒅𝒙

28+9+11+⋯………………
36.
15

324
15

= 21.6
=22 (3)

37.28 (4)
38. (3)
15+1
39. 22
2

9
=8 (3)

477500
40. x 100 = 109.77 (1)
435000

P2000 Q2006 P2009 Q2009 Pocx206 Po5xq P11xq08 P011xq11


x 50 5000 55 5500 250000 275000 302500
y 60 2000 65 2000 120000 120000 130000
z 80 500 90 500 40000 40000 45000
435000 477500

10
QMI 1500 OCTOBER / NOVEMBER 2011

𝑥 𝑦
1. 7
+2

𝟐𝒙+𝟕𝒚
𝟕𝒙
(2)

114
2. 3205 x 100
=3653.7

Difference in price = 3653.7 – 3320


= 333.7 (4)

1 3 2 1
3. - + (3 - )
5 4 5 3

4−15 6 2
+ -
20 5 15

11 18−2
- 20
+ 15

11 16
-20 + 15

𝟏𝟔 𝟏𝟏
𝟏𝟓
- 𝟐𝟎 (5)

4. (1 )

3
5. 4
kg 750grams

750
3
= 250x2 (2)
𝟓𝟎𝟎𝒈𝒓𝒂𝒎𝒔

1
3√64 √500 1
6. 2−3
+ 2
+
√9+16

(64)1/2 ) √5𝑥100 1
1 + 2
+
3 √25
2

10√5 1
8 (4) 2
+5

𝟏𝟎√𝟓 𝟏
32 + 𝟐
+𝟓 (5)

7. (40x40) cm – (25x30) cm
1600cm – 750cm2
850 (5)

8. 3x + 3y = 111.25
2x + 4y = 100.00 (1 )

−6−4
9. gradient =
0− −2

−10
=
2

=-5

Y = = -5x -6b Intercept = (0, -6) (3)

𝟐𝟏
10. 3 = 𝒎+𝟓 (4)

11. y = -2x – 1

Gradient = -2

-4x + 2y + 1 = 0

2y = -4x – 1
4 1
Y=- 2
x-2

𝟏
Y = - 2x - 𝟐 (3)

2
12. x2 , y 2 x 1 y1

( 0, 4 ) (-2 , 0)
4−0
0− −2

4
2

=2 (3)

2𝑥
13. 2 (x + 3) – 5 = +2
3

2
2x + 6 – 5 = 3x + 2

2
2x - x = 2 – 1
3

4𝑥
=1
3

4𝑥 3
4
=4

𝟑
X= (4)
𝟒

14. (3)

15. 1 x 2x +3y = 40

3 x x+y = 17

2x + 3y = 40

3x + 3y = 51

-x = -11

X = 11

3
11 +y = 17

Y = 17 -11

Y=6 (1 )

24. p = r [ ( 1+𝑖 )𝑛−1


𝑖 (1+𝑖)𝑛
]
0.105 (215𝑥12)− 1
(1+
12
22000 = r 0.105 0.105
(1+ )(2.5 𝑥12)
12 12

22000 = 26.28r
220000
R= 26.22

R = 836.97 (3)

25. s = p (1+r)t

= 9000 (1 + 011 ) 3
4

== 9692.0156 – 2320

P = 7372 1i015
011
=7372 ( 1 + )6
4

= 8549,277
(1+𝑖 )𝑛−1
26. s = r 𝑖

0.125 ) (2 𝑥 12)
1+
2
= 2000 0.125
2

= 137 102.22 (5)

27. 15000 – (6460.67 – 2100 )

= 10639.53 (4)

4
28. 0.14 x 10639.03

=1489.46 (2)

10000000
29. 83.25

= 12012 (2)

18000 – 12012

= 5987.98

= 5988 (2)

(158𝑥12)+ (52𝑥15.5)+ (30𝑥13)+ (24𝑥25.4)


30. x 100
(120𝑥10.5)+ (41𝑥13)+ (25𝑥11)+ (21𝑥21)

𝟑𝟕𝟎𝟏.𝟔
x 100 = 147.53 (1)
𝟐𝟓𝟏𝟒.𝟐𝟓

140000
31. 140 000 100 x 100 = 140 000
100

151200
1.08 (14 0000 ) 151 200 10315 x 100 = 146086.96
10313

440+500+380+⋯…………..
32. 10

4620
= 10

= 462 (3)

Receipts 440, 380, 420, 400, 390, 460. 6 receipts

3 (𝑛+1 )
33. Q3 = 4
380, 390, 400, 420, 440, 460, 470, 500, 500, 660

3
= 4 (10 +1)

5
3
= 4 (11)

= 8. 25 observation

= 50010 (4)

34. frequency

1 (2)

35. (2 )

36. mode = 25 range = 62 – 12 = 50 (3)

37. (1)

38. ( 276 – 278 )2 + (278 - 278 )2 + (285 - 278 )2 + (280-278)2 + (277 -278)2 + (282 – 2782) + (268 –
278 )2

= 174 (3)

39. p (x) = r (x) – c ( x)

= 3x – (120 + 0.015x2)

= 3x – 120 + 0.015x2

= 3x – 120 – 0.015x2

= -0.015 x 2 + 3x – 120 (2)


𝑑
40. = - 0.015x2 + 3x – 120
𝑑𝑥

0 - 0.03x + 3
0.03x 3
0.03
= 0.03

X = 100 (1)

6
7
ASSIGNMENT 1 (COMPULSORY)

Due Date Unique Number


04 August 2017 767179

Submit your answers online through myUnisa. No extensions will be granted for submission
of this assignment. NO manual or posted submissions will be allowed.

Question 1

The expression: 𝑥12 𝑓(𝑥1 ) + 𝑥22 𝑓(𝑥2 ) + 𝑥32 𝑓(𝑥3 ) + 𝑥42 𝑓(𝑥4 ) + 𝑥52 𝑓(𝑥5 ) can be written as a
summation notation as:

1) ∑5𝑖=1 𝑥52 𝑓(𝑥5 )


2) ∑5𝑖=1 𝑥12 𝑓(𝑥5 )
3) ∑5𝑖=1 𝑥𝑖2 𝑓(𝑥𝑖 )
4) ∑5𝑖=1 𝑥 2 𝑓(𝑥)
5) None of the above

Question 2

The cost of fish has increased in the ratio of 9 : 7. If the original cost was R5.60 per kg, what
is the new price?

1) R4.35
2) R7.20
3) R50.40
4) R39.50
5) None of the above

Question 3

Six men and eight women have volunteered to serve on a committee. How many different
committees can be formed containing three men and three women?

1) 806 400
2) 336 000
3) 403 200
4) 224 000
5) None of the above
Question 4

A car dealer is offering any 4 of 6 special options at the same price on a specially equipped
car being sold. How many different choices of specially equipped cars do you have?

1) 15
2) 30
3) 120
4) 12
5) None of the above

Question 5

A horizontal drinking trough for cattle is in the shape of a triangular prism. It is 6 m long and
a cross section is in the shape of a triangle with base length 30 cm and height 20 cm. What is
the capacity of the tank in litres?

1) 360 litres
2) 180 litres
3) 36 litres
4) 18 litres
5) None of the above
Question 6

Triangle ABC is similar to triangle ADE. DE = 6 cm and BC = 12 cm. If the area of triangle ABC
is 90 cm2, what is the area of triangle ADE?

1) 90cm2
2) 15cm2
3) 22.5cm2
4) 45cm2
5) None of the above

Question 7

An inspector visits a large company to check its vehicles. The company has the following
vehicles:

5 large-load vehicles; 130 light vans; and 25 cars.

The inspector decides to take a sample of 20% of all the vehicles. Each type of vehicle is to
be represented in the sample; so he will use the stratified random sampling technique. The
number of light vans that should be inspected is:

1) 16
2) 5
3) 32
4) 26
5) None of the above
Questions 8-11 are based on the following information:

Suppose a company has 10 employees, 1 earning R160 000, 1 earning R120 000, 2 earning
R60 000, 1 earning R40 000, and 5 earning R32 000.

Question 8

What is the mean salary for the company?

1) R40 000
2) R36 000
3) R60 000
4) R32 000
5) None of the above

Question 9

What is the median salary?

1) R40 000
2) R36 000
3) R60 000
4) R32 000
5) None of the above

Question 10

What is the mode of the salaries?

1) R40 000
2) R36 000
3) R60 000
4) R32 000
5) None of the above

Question 11

What is the standard deviation of the salaries?

1) R40 000.00
2) R44 621.87
3) R38 416.55
4) R41 583.12
5) None of the above
Questions 12 to 15 are based on the following information.

The incomes (in rands) of seven drivers during the week are: 1 080; 2 000; 1 580; 1 540; 2
500; 1 800; 1 580.

Question 12

The average income is:

1) 1 905.71
2) 1 580.00
3) 586.94
4) 460.00
5) None of the above

Question 13

The mode of the income is:

1) 1 905.71
2) 1 580.00
3) 586.94
4) 460.00
5) None of the above

Question 14

The standard deviation of the income is:

1) 1905.71
2) 460.00
3) 400.00
4) 586.94
5) None of the above

Question 15

The quartile deviation of the income is:

1) 586.94
2) 400.00
3) 1 905.71
4) 460.00
5) None of the above
Question 16 – 17 are based on the following information:

An index of clothing prices for 2005 based on 1998 is to be constructed. The clothing items
considered are shoes and dresses. The information for prices and quantities for both years is
given below. Use 1998 as the base period and 100 as the base value.

Question 16

Determine Laspeyres price index.

1) 98.9
2) 103.7
3) 112.9
4) 106.4
5) None of the above

Question 17

Determine the Paasche price index.

1) 98.9
2) 103.7
3) 112.9
4) 106.4
5) None of the above
Question 18

The number of items produced by a company for 1999 and 2005 and the wholesale prices
for the two periods are:

Find the index of the value of production for 2005 using 1999 as the base period.

1) 99.7
2) 127.1
3) 100.3
4) 110.6
5) None of the above

Question 19

Suppose the Consumer Price Index for the latest month is 134.0 (1992 as 100). What is the
purchasing power of the rand?

1) R75.00
2) R0.075
3) R0.75
4) R7.50
5) None of the above

Question 20

Mrs Mthombeni inherits some money. She intends to spend R25 000 on a holiday in Mexico.
How much money will this amount be in Mexican pesos (MXN)? Use the exchange rate:

1.6915MXN = R1.00

The correct answer is:

1) 14 799.78MXN
2) 25 001.69MXN
3) 34 799.78MXN
4) 42 287.50MXN
5) None of the above
Question 21

Assume that in the year 2014 the Consumer Price index (CPI) was 102.7 in February and
110.5 in November. An employee’s wage was R20 000 in February and R22 145 in
November. In relation to the value of the rand in November, his wage has:

1) Increased by R2 145.00
2) Decreased by R566.52
3) Decreased by R395.15
4) Increased by R566.52
5) None of the above
ASSIGNMENT 2 (COMPULSORY)

Due Date Unique Number


25 August 2017 753582

Submit your answers online through myUnisa. No extensions will be granted for submission
of this assignment. NO manual or posted submissions will be allowed.

Question 1

An electric utility company determines the monthly bill for a residential customer by adding an
energy charge of 8.38 cents per kilowatt hour to its base charge of R4.95 per month. Write an
equation for the monthly charge 𝑦 (in rands) in terms of 𝑥, the number of kilowatt-hours used.

1) 𝑦 = 8.38𝑥 + 4.95
2) 𝑦 = 0.0838𝑥 + 4.95
3) 𝑦 = 0.838𝑥 + 4.95
4) 𝑦 = 0.0838𝑥 + 495
5) None of the above

Question 2

Census Data from 2003 for various age groups show that for each R100 increase in the median
weekly income for men, the median weekly income of women increases by R61.90. Also, for workers
of ages 25 to 54 the median weekly income for men was R676 and for women was R527. Let M
represent the median weekly income for men and W the median weekly income for women, and
write the equation of the line that gives 𝑊 as a linear function of 𝑀.

1) 𝑊 = 0.381𝑀 + 269.444
2) 𝑊 = 2.625𝑀 − 1247.278
3) 𝑊 = 0.619𝑀 + 108.556
4) 𝑊 = 1.6166𝑀 − 565.084
5) None of the above

Questions 3 to 4 refer to the following information:


1
Consider the function 𝑦 = 12𝑥 − 𝑥 2 and its graph.
2

Question 3

Determine the vertex of the graph.

1) (−12; −216)
2) (24; 0)
3) (12; 72)
4) (√24; 0)
5) None of the above
Question 4

Determine the x-intercepts of the graph.

1) (0; 0) and (−12; −216)


2) (0; 0) and (24; 0)
3) (0; 0) and (12; 72)
4) (0; 0) and (√24; 0))
5) None of the above

Questions 5 to 6 are on the following graphs below.

Question 5

Determine the vertex of each graph.

1) 𝑓1 (𝑥) − (3; 16)𝑎𝑛𝑑 𝑓2 (𝑥) − (3; 1)


2) 𝑓1 (𝑥) − (0; 7)𝑎𝑛𝑑 𝑓2 (𝑥) − (0; 10)
3) 𝑓1 (𝑥) − (−1; 0)𝑎𝑛𝑑 𝑓2 (𝑥) − (7; 0)
4) 𝑓1 (𝑥) − (1; 3)𝑎𝑛𝑑 𝑓2 (𝑥) − (0; 0)
5) None of the above

Question 6

Determine the y-intercepts of each graph.

1) 𝑓1 (𝑥) − (3; 16)𝑎𝑛𝑑 𝑓2 (𝑥) − (3; 1)


2) 𝑓1 (𝑥) − (0; 7)𝑎𝑛𝑑 𝑓2 (𝑥) − (0; 10)
3) 𝑓1 (𝑥) − (−1; 0)𝑎𝑛𝑑 𝑓2 (𝑥) − (7; 0)
4) 𝑓1 (𝑥) − (1; 3)𝑎𝑛𝑑 𝑓2 (𝑥) − (0; 0)
5) None of the above
Question 7-8 are based on the following information:

If P rand is invested for t years at interest rate r, compounded continuously, then the future value of
the investment is given by 𝑆 = 𝑃𝑒 𝑟𝑡

Question 7

Express the doubling time (the time it takes to double, i.e. S = 2P) of this investment in logarithmic
format.

ln 2
1) 𝑡 = 𝑟
−2 ln 2
2) 𝑡 =
ln 𝑟
2 log 2
3) 𝑡 = 𝑟
4) 𝑡 = −2 log 0.5𝑟
5) None of the above

Question 8

If the investment earns 10% compounded continuously, in how many years will it double?

1) 4.604 years
2) 6.93 years
3) 6.02 years
4) 2.602 years
5) None of the above

Question 9

Jenny has R90 000 to invest. She has chosen one relatively safe investment fund that has an annual
yield of 10% and another, riskier one that has a 15% annual yield. How much should she invest in
each fund if she would like to earn R10 000 in one year from her investments?

1) R20 000 at 10% and R70 000 at 15%


2) R70 000 at 10% and R20 000 at 15%
3) R50 000 at 10% and R40 000 at 15%
4) R40 000 at 10% and R50 000 at 15%
5) None of the above

Question 10

A nurse has two solutions that contain different concentrations of a certain medication. One is a
12.5% concentration and the other is a 5% concentration. How many millilitres (ml) of each should
she mix to obtain 20ml of an 8% concentration?

1) 4ml at 12.5% and 16ml at 5%


2) 16ml at 12.5% and 4ml at 5%
3) 8ml at 12.5% and 12ml at 5%
4) 12ml at 12.5% and 8ml at 5%
5) None of the above

Questions 11 to 12 refer to the information below:

A group of wholesalers will buy 50 dryers per month if the price is R200 and 30 per month if the
price is R300. The manufacturer is willing to supply 20 if the price is R210 and 30 if the price is R230.
Assume that the resulting supply and demand functions are linear.

Question 11

Find the equilibrium point (where demand equals supply) for the market.

1) Quantity is 30 and Price is R265


2) Quantity is 35 and Price is R275
3) Quantity is 40 and Price is R250
4) Quantity is 45 and Price is R260
5) None of the above

Question 12

If the wholesaler is taxed R14 per unit sold, what is the new equilibrium point? (Hint: the tax amount
affects the supply of the units, and not the demand of the units.)

1) Quantity is 32 and Price is R320


2) Quantity is 34 and Price is R280
3) Quantity is 36 and Price is R270
4) Quantity is 38 and Price is R260
5) None of the above

Question 13

When solving the following inequality: −5𝑥 + 3 ≥ 𝑥 − 15, the solution to one decimal place is:

1) 𝑥 ≥ 2.0
2) 𝑥 ≤ 4.5
3) 𝑥 ≤ 3.0
4) 𝑥 ≥ 5.0
5) None of the above

Questions 14 to 15 are based on the information below:

CDF Appliances has assembly plants in Atlanta and Fort Worth, where they produce a variety of
kitchen appliances, including a 12-cup coffee maker and a cappuccino machine. At the Atlanta plant,
160 of the 12-cup models and 200 of the cappuccino machines can be assembled each hour. At the
Fort Worth plant, 800 of the 12-cup models and 200 of the cappuccino machines can be assembled
each hour. CDF Appliances expects orders for at least 64 000 of the 12-cup models and at least 40
000 of the cappuccino machines. At each plant, the number of assembly hours available for these
two appliances is constrained by each plant’s capacity and the need to fill the orders. Let x be the
number of assembly hours at the Atlanta plant, and let y be the number of assembly hours at the Fort
Worth plant.
Question 14

Write the system of inequalities that describes these assembly plant constraints.

1) 160𝑥 + 800𝑦 ≥ 40 000; 200𝑥 + 200𝑦 ≥ 64 000; 𝑥 ≥ 0; 𝑦 ≥ 0


2) 160𝑥 + 800𝑦 ≥ 64 000; 200𝑥 + 200𝑦 ≥ 40 000; 𝑥 ≥ 0; 𝑦 ≥ 0
3) 800𝑥 + 160𝑦 ≥ 40 000; 200𝑥 + 200𝑦 ≥ 64 000; 𝑥 ≥ 0; 𝑦 ≥ 0
4) 800𝑥 + 160𝑦 ≥ 64 000; 200𝑥 + 200𝑦 ≥ 40 000; 𝑥 ≥ 0; 𝑦 ≥ 0
5) None of the above

Question 15

For CDF appliances to meet the expected orders, what is the minimum number of hours that needs
to be allocated at each manufacturing plant? (Tip: Solve the system of inequalities and identify the
extreme point(s) of the region of solution).

1) 150 hours at the Atlanta plant and 150 hours at the Fort Worth plant.
2) 150 hours at the Atlanta plant and 50 hours at the Fort Worth plant.
3) 70 hours at the Atlanta plant and 100 hours at the Fort Worth plant.
4) 80 hours at the Atlanta plant and 90 hours at the Fort Worth plant.
5) None of the above.

Question 16

A company manufactures two types of chairs, standard and plush. Standard chairs require 2 hours to
construct and finish, and plush chairs require 3 hours to construct and finish. Upholstering takes 1
hour for standard chairs and 3 hours for plush chairs. There are 240 hours per day available for
construction and finishing, and 150 hours per day are available for upholstering. Which of the
following is NOT an extreme point of the solution space of the problem described above?

Let x be the number of standard chairs produced each day, and let y be the number of plush chairs
produced.

1) (0; 50)
2) (90; 20)
3) (105; 10)
4) (120; 0)
5) None of the above
Questions 17 and 18 are based on the information below:

A firm sells its product for R200 per unit. The cost per unit (per month) is 80 + 𝑥, where x represents
the number of units sold per month.

Question 17

Define the marginal profit function.

1) 𝑀𝑃(𝑥) = 120 − 2𝑥
2) 𝑀𝑃(𝑥) = 80 − 2𝑥
3) 𝑀𝑃(𝑥) = 120𝑥 − 80
4) 𝑀𝑃(𝑥) = 199𝑥 − 80
5) None of the above

Question 18

What is the marginal profit at a production level of 20 units?

1) R80
2) R40
3) R2 320
4) R3 900
5) None of the above

Questions 19 and 20 are based on the information below:

The daily total cost in rand for a certain factory to produce 𝑥 kitchen blenders is given by 𝐶(𝑥) =
0.001𝑥 3 − 0.3𝑥 2 + 32𝑥 + 2 500.

Question 19

Define the marginal cost function for these kitchen blenders.

1) 𝑀𝐶 = 0.003𝑥 2 + 0.6𝑥 + 32
2) 𝑀𝐶 = 0.001𝑥 2 − 0.3𝑥 + 32
3) 𝑀𝐶 = 0.003𝑥 2 − 0.6𝑥 + 32
4) 𝑀𝐶 = 0.002𝑥 2 + 0.3𝑥 + 32
5) None of the above

Question 20

Find the marginal cost when 80 units are produced.

1) R36.00 per unit


2) R3.20 per unit
3) R99.20 per unit
4) R68.80 per unit
5) None of the above
ASSIGNMENT 3 (COMPULSORY)

Due Date Unique Number


18 September 2017 743987

Submit your answers online through myUnisa. No extensions will be granted for submission
of this assignment. NO manual or posted submissions will be allowed.

Question 1

If R2 000 is borrowed for one-half year at a simple interest rate of 12% per year, what is the future
value of the loan at the end of the half-year?

1) R1 785.71
2) R1 886.79
3) R2 120.00
4) R2 240.00
5) None of the above

Question 2

Thabo bought Gijima-Electric stock for R6 125.00 and after 6 months, the value of her shares had
risen by R138.00 and dividends totalling R144.14 had been paid. Find the simple interest rate she
earned on this investment if she sold the stock at the end of the 6 months.

1) 4.50%
2) 4.61%
3) 4.70%
4) 9.21%
5) None of the above

Question 3

If R8 000 is invested at 6% simple interest for 9 months, find the future value of the investment.

1) R8 240
2) R8 360
3) R8 480
4) R8 720
5) None of the above

Question 4

If R3 000 is invested for 4 years at 9% compounded annually, how much interest is earned?
1) R1 080.00
2) R1 234.74
3) R1 294.22
4) R1 299.79
5) None of the above
Question 5

What amount must be invested at 6.5%, compounded quartely, so that it will be worth R25 000 after
8 years?
1) R14 883.01
2) R14 925.27
3) R14 986.46
4) R14 863.01
5) None of the above
Question 6

If R5 000 is invested at 6% compounded monthly for 5 years, find the future value of the investment.
1) R6 719.58
2) R6 749.13
3) R6 744.25
4) R6 749.29
5) None of the above

Question 7

How long does it take an investment of R10 000 to double if it is invested at 8%, compounded daily?
1) 8.70 years
2) 8.69 years
3) 8.67 years
4) 8.83 years
5) None of the above

Question 8

Jenny invests R2 000 at the end of each of 8 years in an account that earns 10%, compounded
annually. After the initial 8 years, no additional contributions are made, but the investment
continues to earn 10%, compounded annually, for 36 more years (until Jenny is age 65). How much
does Jenny have at age 65?
1) R530 112.61
2) R707 028.03
3) R1 305 281.52
4) R598 253.61
5) None of the above

Question 9

Suppose R100 is deposited at the end of each month for 3 years in an account that pays 6%,
compounded monthly. Find the future value.
1) R4 287.66
2) R3 933.61
3) R4 038.05
4) R3 953.28
5) None of the above
Question 10

A small business invests R10 000 at the end of each month in an account that earns 6% compounded
monthly. How long will it take until the business has R1 000 000 toward the purchase of its own
office building? (Round your answer up to the next integer)

1) 79 months
2) 82 months
3) 77 months
4) 84 months
5) None of the above

Question 11

Find the lump sum that one must invest in an annuity in order to receive R1 000 at the end of each
month for the next 16 years, if the annuity pays 9%, compounded monthly.
1) R151 603.71
2) R99 750.72
3) R101 572.77
4) R426 410.43
5) None of the above

Question 12

Find the present value of an annuity that pays R2 000 at the end of each 3-month period for 3.5
years and money is worth 4%, compounded quarterly.
1) R26 007.41
2) R29 428.12
3) R32 185.28
4) R29 894.84
5) None of the above

Question 13

What lump sum will be needed to generate payments of R5 000 at the end of each quarter for a
period of 5 years if money is worth 7%, compounded quarterly?
1) R87 774.26
2) R85 230.28
3) R83 764.41
4) R82 003.95
5) None of the above

Question 14

Jack and Jill have R30 000 for a down payment, and their budget can accommodate a monthly
mortgage payment of R850.00. What is the most expensive home they can buy if they can borrow
money for 30 years at 7.8%, compounded monthly?
1) R131 378.02
2) R147 030.62
3) R133 725.88
4) R148 076.79
5) None of the above
Question 15

Find the amount of each payment if a debt of R25 000 is to be amortized with equal quarterly
payments over 6 years, and money is worth 7%, compounded quarterly.
1) R847.14
2) R873.72
3) R1 284.64
4) R1 311.22
5) None of the above

Question 16

A new college graduate determines that she can afford a car payment of R350 per month. If the car
manufacturer is offering a special 2.1% financing rate, compounded monthly for 4 years, how much
can she borrow and still have a R350 monthly payment?
1) R15 953.73
2) R16 100.25
3) R17 336.65
4) R17 509.81
5) None of the above

Question 17

A 42-month car loan has monthly payments of R411.35. If the interest rate is 8.1%, compounded
monthly, find the unpaid balance immediately after the 24th payment.
1) R4 295.12
2) R6 950.13
3) R3 828.53
4) R9 086.01
5) None of the above

Questions 18 to 20 are based on the information below:

A man buys a house for R200 000. He makes a R50 000 down payment and agrees to amortize the
rest of the debt with quarterly payments over the next 10 years. Interest on the debt is 12%,
compounded quarterly.

Question 18

What is the size of the quarterly payments?


1) R6 489.36
2) R6 636.91
3) R8 652.48
4) R8 849.21
5) None of the above

Question 19
What is the total amount of the payments?
1) R346 099.20
2) R265 476.40
3) R259 574.40
4) R353 968.40
5) None of the above

Question 20
What is the total amount of interest paid?
1) R146 099.20
2) R109 574.40
3) R153 964.40
4) R115 476.40
5) None of the above
Question 1: Example
Consider the following sequence:
𝒙𝟏 𝒚𝟏 + 𝒙𝟐 𝒚𝟐 + 𝒙𝟑 𝒚𝟑 + 𝒙𝟒 𝒚𝟒 + 𝒙𝟓 𝒚𝟓
Write it in summation notation.
Answer
Tips:
• First notice that the x and y subscripts change from 1 to 5 in the sequence.
• Also notice that the subscript is the only thing that changes in the sequence while all
else remains the same
𝒏
• Writing something in summation notation means using the summation symbol 𝒊+𝟏 𝒙𝒊 or
𝒏 𝒏
𝒊+𝟎 𝒙𝒊 or 𝒊+-𝟏 𝒙𝒊 depending on what changing number/subscript your given sequence
starts from.

ü So we already know that our “i” starts at 1 and ends at 5. Meaning our symbol will look
something like this:
𝟓
𝒊+𝟏 𝒙𝒊

ü From the summation symbol we know that the “i” represents the subscripts that are
changing meaning that the general term of our sequence is 𝒙𝒊 𝒚𝒊
ü Putting these two together, we finally get:
𝟓

𝒙𝒊 𝒚𝒊 𝒊
𝒊+𝟏

Question 2: Example
The number of girls studying at UNISA in second semester has increased in the ratio 3:2. If
the number of girls at the start of first semester were only 20, how many of girls do we have
now?
Answer
Tips:
• Another way of writing a ratio is as a fraction.

𝟓
e.g. 𝟓 ∶ 𝟐 → = 𝟐. 𝟓
𝟐

• Sometimes they can give the ratio as words:


𝟓
e.g. the ratio of 5 buttons to 2 shirts → 𝟓 ∶ 𝟐 → = 𝟐. 𝟓
𝟐

ü Convert your ratio to a fraction:


𝟑
𝟑∶𝟐→ = 𝟏. 𝟓
𝟐
ü The question says the number has increased by this ratio. When something increases by
something, we must multiply the two:
So we get: 𝟐𝟎×𝟏. 𝟓 = 𝟑𝟎

The number of girls in second semester has now increased to 30.

Question 3: Example
How many ways are there to select a committee to develop a discrete mathematics course at a
school if the committee is to consist of 3 faculty members from the mathematics department
and 4 from the computer science(CS) department, if there are 9 faculty members of the math
department and 11 of the CS department?
Answer
Tips:
• First consider if the order is important, if it is then we know to use a permutation and if
it is not then we know to use a combination.
• Consider the number of ways to choose the faculty members from the mathematics
department, and then consider the number of ways to choose the faculty members from
the CS department

ü We can pick the 3 faculty members out of the total 9 from the maths department as
follows:
𝟗!
𝒙𝟗 𝑪𝟑 = = 𝟖𝟒
𝟗 − 𝟑 !×𝟑!
ü We can pick the 4 faculty members out of the total 11 from the CS department as
follows:
𝟏𝟏!
𝒙𝟏𝟏 𝑪𝟒 = = 𝟑𝟑𝟎
𝟏𝟏 − 𝟒 !×𝟒!

Therefore, there are:


𝟗! 𝟏𝟏!
𝒙𝟗 𝑪𝟑 ×𝒙𝟏𝟏 𝑪𝟒 = × = 𝟖𝟒×𝟑𝟑𝟎 = 𝟐𝟕𝟕𝟐𝟎 𝒘𝒂𝒚𝒔
𝟗 − 𝟑 !×𝟑! 𝟏𝟏 − 𝟒 !×𝟒!
Question 4: Example
On each delivery, a postman delivers letters to 3 of the 12 suburbs in his territory. In how
many different ways can he schedule his route?
Answer
Tips:
• The order in which he delivers to the suburbs is not mentioned, meaning that this is a
combination

ü The number of ways in which he can schedule his route is:

𝟏𝟐! 𝟏𝟐!
𝒙𝟏𝟐 𝑪𝟑 = × = 𝟐𝟐𝟎 𝒘𝒂𝒚𝒔
𝟏𝟐 − 𝟑 !×𝟑! 𝟗!×𝟑!

Question 5 & 6: Examples


Please see the hints posted on the main QMI1500 site posted by the lecturer before
proceeding to the examples below.
Example 1
Find the volume of the triangular prism shown in the diagram:

Solution:
ü 𝑽𝒐𝒍𝒖𝒎𝒆 𝒐𝒇 𝒂 𝒑𝒓𝒊𝒔𝒎 = 𝑨𝒓𝒆𝒂 𝒐𝒇 𝒃𝒂𝒔𝒆 ×𝒉𝒆𝒊𝒈𝒉𝒕
ü The base is a triangle, so we must find the area of the triangle which is:
𝟏 𝟏
𝑨= ×𝒃𝒂𝒔𝒆×𝒉𝒆𝒊𝒈𝒉𝒕 = ×𝟗×𝟏𝟐 = 𝟓𝟒𝒄𝒎𝟐
𝟐 𝟐

Hence,
𝑽 = 𝑨×𝒉 = 𝟓𝟒×𝟏𝟖 = 𝟗𝟕𝟐𝒄𝒎𝟑
Example 2
Consider the following two similar triangles:

If 𝐴𝐵 = 6, 𝐷𝐸 = 3 𝑎𝑛𝑑 𝑡ℎ𝑒 𝑎𝑟𝑒𝑎 𝑜𝑓 𝐷𝐸𝐹 = 6𝑐𝑚_ . What is the area of ABC


Solution:
ü The ratio of the given sides is:

𝑨𝑩 𝟔
= =𝟐
𝑫𝑬 𝟑
ü If you follow the hints posted by the lecturer you will find out that if ratio of the sides of
two similar triangles is 𝒙 then the ratio of the areas of the triangles is 𝒙𝟐

Therefore,
𝑨𝒓𝒆𝒂 𝒐𝒇 𝑨𝑩𝑪 = 𝟐𝟐 ×𝑨𝒓𝒆𝒂 𝒐𝒇 𝑫𝑬𝑭
= 𝟒×𝟔 = 𝟐𝟒𝒄𝒎𝟐

Question 7: Example
There are 3 grade 12 classes at John’s high school, class one has 50 students, class two has 60
𝟏
students and class 3 has 80 students. An examiner decides to take a sample of of the entire
𝟐

grade 12 to test the new syllabus. All classes must be represented in the sample. Show how
many students will be sampled in each class
Answer
ü 𝑻𝒐𝒕𝒂𝒍 𝒈𝒓𝒂𝒅𝒆 𝟏𝟐 𝒔𝒕𝒖𝒅𝒆𝒏𝒕𝒔 = 𝟓𝟎 + 𝟔𝟎 + 𝟖𝟎 = 𝟏𝟗𝟎
ü 𝑷𝒓𝒐𝒑𝒐𝒓𝒕𝒊𝒐𝒏𝒂𝒍 𝒔𝒂𝒎𝒑𝒍𝒆 𝒔𝒊𝒛𝒆 = 𝟏𝟗𝟎×𝟎. 𝟓 = 𝟗𝟓
ü Representation of each class will be:
𝟓𝟎
𝑪𝒍𝒂𝒔𝒔 𝟏 = ×𝟗𝟓 = 𝟐𝟓
𝟏𝟗𝟎
𝟔𝟎
𝑪𝒍𝒂𝒔𝒔 𝟐 = ×𝟗𝟓 = 𝟑𝟎
𝟏𝟗𝟎
𝟖𝟎
𝑪𝒍𝒂𝒔𝒔 𝟑 = ×𝟗𝟓 = 𝟒𝟎
𝟏𝟗𝟎
ü We can check if this is correct by adding the values:
𝟐𝟓 + 𝟑𝟎 + 𝟒𝟎 = 𝟗𝟓
ü This is 0.5 of grade 12.

Question 8-15 Example


The manager of Pies for a Palace keeps record of the number of each type of pie sold each
day. The number of pepper steak pies sold for the past 7 days is:
Day 1 2 3 4 5 6 7
Number 26 35 45 28 73 52 37
Calculate the following:
1. Average
2. Median
3. Mode
4. Standard deviation
5. Quartile deviation

Answer
Tip: Always remember to arrange the data in ascending order (from small to large)

ü 26 28 35 37 45 52 73
1. The average is also called the mean. It is calculated as:

𝒙
𝒙=
𝒏
→ 𝑤ℎ𝑒𝑟𝑒 𝑥 𝑖𝑠 𝑡ℎ𝑒 𝑠𝑢𝑚 𝑜𝑓 𝑎𝑙𝑙 𝑦𝑜𝑢𝑟 𝑜𝑏𝑠𝑒𝑟𝑣𝑎𝑡𝑖𝑜𝑛𝑠 𝑑𝑎𝑡𝑎

→ 𝑎𝑛𝑑 𝑛 𝑡ℎ𝑒 𝑛𝑢𝑚𝑏𝑒𝑟 𝑜𝑓 𝑦𝑜𝑢𝑟 𝑜𝑏𝑠𝑒𝑟𝑣𝑎𝑡𝑖𝑜𝑛𝑠


𝒙 𝟐𝟔 + 𝟐𝟖 + 𝟑𝟓 + 𝟑𝟕 + 𝟒𝟓 + 𝟓𝟐 + 𝟕𝟑
𝒙= =
𝒏 𝟕
𝟐𝟗𝟔
= = 𝟒𝟐. 𝟐𝟗
𝟕
2. Determine the position of the median first:
- There are 7 values in the dataset, thus n =7. The position of the median is
determined as:
𝒏+𝟏 𝟖
= =𝟒
𝟐 𝟐
- The median is the value in the 4th position after arranging the data. Count up to
value number 4 on the data.
𝑽𝒂𝒍𝒖𝒆 𝒏𝒖𝒎𝒃𝒆𝒓 𝟒 = 𝟑𝟕
- The median is therefore 37.
3. The mode is the value that appears most often. This data set does not have a mode.
4. The standard deviation is the square root of the variance, given by:

(𝒙 − 𝒙)𝟐 (𝟐𝟔 − 𝟒𝟐. 𝟐𝟗)𝟐 + (𝟐𝟖 − 𝟒𝟐. 𝟐𝟗)𝟐 + ⋯ + (𝟕𝟑 − 𝟒𝟐. 𝟐𝟗)𝟐
𝑺𝟐 = =
𝒏−𝟏 𝟕−𝟏
𝟏𝟓𝟗𝟓. 𝟒𝟑
= = 𝟐𝟔𝟓. 𝟗𝟎
𝟔
Hence, standard deviation = 𝑺𝟐 = 𝟐𝟔𝟓. 𝟗𝟎 = 𝟏𝟔. 𝟑𝟎
5. The quartile deviation is given by:
𝑸𝟑 − 𝑸𝟏
𝑸𝑫 =
𝟐
𝟏 𝟑
𝑤ℎ𝑒𝑟𝑒 𝑸𝟏 = 𝒏 + 𝟏 𝒂𝒏𝒅 𝑸𝟑 = (𝒏 + 𝟏)
𝟒 𝟒
𝟏 𝟏 𝟖
→ 𝑸𝟏 = 𝒏 + 𝟏 = 𝟕 + 𝟏 = = 𝟐
𝟒 𝟒 𝟒
𝟑 𝟑 𝟐𝟒
→ 𝑸𝟑 = 𝒏 + 𝟏 = 𝟕 + 𝟏 = =𝟔
𝟒 𝟒 𝟒
- The first quartile is the value in the 2nd position after arrangement and the third
quartile is the value in the 6th position. Count up to value number 2 and 4 on the
data.

→ 𝑽𝒂𝒍𝒖𝒆 𝒏𝒖𝒎𝒃𝒆𝒓 𝟐 = 𝟐𝟖
→ 𝑽𝒂𝒍𝒖𝒆 𝒏𝒖𝒎𝒃𝒆𝒓 𝟔 = 𝟓𝟐
𝑸𝟑 − 𝑸𝟏 𝟓𝟐 − 𝟐𝟖
𝐻𝑒𝑛𝑐𝑒, 𝑸𝑫 = = = 𝟏𝟐
𝟐 𝟐
Question 16 and 17 example
The City Ice and Beverage Store sells a complete line of beer, wine and soft drink products.
Listed below are the quantities sold and the prices for each beverage for 2008 and 2010.
Item 2008 2010
Price Quantity Price Quantity
Beer 19.00 6000 21.50 5000
Wine 32.00 3000 35.00 5000
Soft drinks 6.00 9000 7.50 11000
Calculate the the Paasche price index and the Laspeyres price index for 2010 with 2008 as the
base year.
Solution
Item
𝒑𝟎 ×𝒒𝒏 𝒑𝒏 ×𝒒𝒏 𝒑𝒏 ×𝒒𝟎 𝒑𝟎 ×𝒒𝟎
𝒑𝟐𝟎𝟎𝟖 ×𝒒𝟐𝟎𝟏𝟎 𝒑𝟐𝟎𝟏𝟎 ×𝒒𝟐𝟎𝟏𝟎 𝒑𝟐𝟎𝟏𝟎 ×𝒒𝟐𝟎𝟎𝟖 𝒑𝟐𝟎𝟎𝟖 ×𝒒𝟐𝟎𝟎𝟖

Beer 19 x 5000 = 21.5 x 5000 = 21.50 x 6000 19 x 6000 =


95000 107500 = 129000 114000
Wine 32 x 5000 = 35 x 5000 = 35 x 3000 = 32 x 3000 = 96000
160000 175000 105000
Soft 6 x 11000 = 7.5 x 11000 = 7.5 x 9000 = 6 x 9000 = 54000
drinks 66000 82500 67500
Total 𝒑𝟎 𝒒𝒏 𝒑𝒏 𝒒𝒏 𝒑𝒏 𝒒𝟎 𝒑𝟎 𝒒𝟎

= 𝟑𝟐𝟏𝟎𝟎𝟎 = 𝟑𝟔𝟓𝟎𝟎𝟎 = 𝟑𝟎𝟏𝟓𝟎𝟎 = 𝟐𝟔𝟒𝟎𝟎𝟎

𝒑𝒏 𝒒𝒏 𝟑𝟔𝟓𝟎𝟎𝟎
The Paasche price index = ×𝟏𝟎𝟎 = ×𝟏𝟎𝟎 = 𝟏𝟏𝟑. 𝟕𝟏
𝒑𝟎 𝒒𝒏 𝟑𝟐𝟏𝟎𝟎𝟎

𝒑𝒏 𝒒𝟎 𝟑𝟎𝟏𝟓𝟎𝟎
The Laspeyres price index = ×𝟏𝟎𝟎 = ×𝟏𝟎𝟎 = 𝟏𝟏𝟒. 𝟐𝟎
𝒑𝟎 𝒒𝟎 𝟐𝟔𝟒𝟎𝟎𝟎
Question 18 example
The number of items produced by Goodness Co. Products for 2000 and 2008 and the
wholesale prices for the two periods are:
Item produced Price (Rands) Number Produced
2000 2008 2000 2008

Laptops 3 4 10 000 9000


Maps 1 5 600 200
Calendars 10 8 3000 5000
Find the index of the value of production for 2008 using 200 as the base period.
Answer
Item 𝒑𝒏 ×𝒒𝒏 𝒑𝟎 ×𝒒𝟎
𝒑𝟐𝟎𝟎𝟖 ×𝒒𝟐𝟎𝟎𝟖 𝒑𝟐𝟎𝟎𝟎 ×𝒒𝟐𝟎𝟎𝟎

Notepads 4×9000 = 36000 3×10000 = 30000


Keyboards 5×200 = 1000 1×600 = 600
Printers 8×5000 = 40000 10×3000 = 30000
Total 𝒑𝒏 𝒒𝒏 = 𝟕𝟕𝟎𝟎𝟎 𝒑𝟎 𝒒𝟎 = 𝟔𝟎𝟔𝟎𝟎
𝒑𝒏 𝒒𝒏 𝟕𝟕𝟎𝟎𝟎
The value index = ×𝟏𝟎𝟎 = ×𝟏𝟎𝟎 = 𝟏𝟐𝟕. 𝟎𝟔
𝒑𝟎 𝒒𝟎 𝟔𝟎𝟔𝟎𝟎

Question 19 example
Suppose the Consumer Price Index this month is 125.0 (2002 as 100). What is the purchasing
power of the dollar?
Answer
The Consumer Price Index (CPI) is used to determine the purchasing power of any currency.
$𝟏
𝑷𝒖𝒓𝒄𝒉𝒂𝒔𝒊𝒏𝒈 𝒑𝒐𝒘𝒆𝒓 𝒐𝒇 𝒕𝒉𝒆 𝒅𝒐𝒍𝒍𝒂𝒓 = 𝟏𝟎𝟎 = $𝟎. 𝟖𝟎
𝟏𝟐𝟓

Question 20 example
At a certain point in time the South African rand (R) is converted to the Hong Kong dollar
(HKD) at a rate of
𝟏. 𝟔𝟕𝑯𝑲𝑫 = 𝑹𝟏. 𝟎𝟎
If Luke has R50 000 when he arrives in Hong Kong, how much will he have to spend in
Hong Kong?
Answer
𝑹𝒂𝒏𝒅 ∶ 𝑯𝑲𝑫
𝟏 ∶ 𝟏. 𝟔𝟕
𝟏 𝟏. 𝟔𝟕

𝟏. 𝟔𝟕 𝟏. 𝟔𝟕
𝟏
:𝟏
𝟏. 𝟔𝟕
𝟏
Hence 1 rand is equal to HKD. This is our exchange rate.
𝟏.𝟔𝟕

Therefore, Luke has:

𝟏
𝟓𝟎𝟎𝟎𝟎× = 𝟐𝟗𝟗𝟒𝟎. 𝟏𝟐𝑯𝑲𝑫
𝟏. 𝟔𝟕

Question 21 example
The take-home pay of Mr Gaza and the CPI for 2003 and 2007 are:

Year Take-Home Pay CPI

2016 R250 000 102.80

2017 R420 000 111.5

(a) What was Mr Gaza’s real income in 2016?


(b) What was his real income in 2017?
(c) Interpret your findings

Answer
Year Real income

(a) 2016 𝟐𝟓𝟎 𝟎𝟎𝟎


𝟏𝟎𝟎 = 𝟐𝟒𝟑 𝟏𝟗𝟎. 𝟔𝟔
𝟏𝟎𝟐. 𝟖𝟎

(b) 2017 𝟒𝟐𝟎 𝟎𝟎𝟎


𝟏𝟎𝟎 = 𝟑𝟕𝟔 𝟔𝟖𝟏. 𝟔𝟏
𝟏𝟏𝟏. 𝟓

(c) Mr Gaza’s take-home pay increased by:


𝟑𝟕𝟔 𝟔𝟖𝟏. 𝟔𝟏 − 𝟐𝟒𝟑 𝟏𝟗𝟎. 𝟔𝟔 = 𝑹 𝟏𝟑𝟑 𝟒𝟗𝟎. 𝟗𝟓
Note the following for question 1 - base charge is constant. There is also an energy charge of 8.38c per
kW. This means that for 1kW produced and used then the charge is (8.38)*(1), for 2 kW produced and
used then the charge is (8.38)*(2)...and so on. So the energy charge varies with the amount of energy
produced and used. This should tell you that the energy charge is one of your variables. Your monthly bill
is the other variable, as it depends on the amount of energy that you have (and will in total include the
base charge, which is constant). From this you should be able determine that the monthly bill is your
dependent variable. From the given information, this is how you would decide which variable will be
represented by x (independent) and which will be presented by y (dependent). This is something that
you are expected to know for the exam. Fortunately, in this example this is told to you and you do not
have to figure it.

What is likely to be throwing you off is the fact that the energy charge is in cents and the base charge is
in Rands. Convert to the same unit and solve the equation

Note for question 2

First Hint:
Don't be thrown off by the use of different letters "w" and "m" instead of "x" and y"…the
concepts are the same as what is covered in the note
So "m" = income data for men
"w" = income data for women

Determine which variable is dependent and which is independent. You know that the form is
y = ax + b, where x is independent and y is dependent. The same will apply here for your
variables "m" and "w". The way the answer options are set up actually gives this away. Apply
what you have learnt

Second Hint:
If you have gone through the exercises in your workbook for this section, you will know that to
calculate the slope of a function of linear function you can use the change in y-values and
the change in x-values. Question 2 gives you this information….use it!

Third and Final Hint:


You have been given a set of co-ordinates in Question 2….use it!

Note for question 3 and 4 - re-arrange your equation so that it is in the standard format of a quadratic
function. From there it should be easy to see what the values are for the constants "a", "b" and "c". The
note that I compiled for you tells you how to find the vertex as well as the x and y intercepts. I also go
through an example of how you do this, step by step...refer to it. If you still have questions on the example
covered in the note then let me know

Note for question 5 and 6 - these graphs are drawn to scale and you therefore do not need equations.
You can read the co-ordinates off the graphs. By this I mean, at the point that you are considering
(whether it is the vertex or an intercept) draw a vertical line from that point to the graph itself and that
will give you the value of x. If you draw a horizontal line from the same point to the graph it will give you
the value of y. The x and y axes are labelled. And always remember that co-ordinates are in the form (x,
y)...always!

Note for question 7 – you are given an equation with 4 variables (S, P, r, and t)
S = accumulated value
P = initial investment
r = interest rate
t = period/time
You need to solve for “t” where the accumulated value equals two times what you initially invested. You
are told that the accumulated value is infact equal to 2P (Where P is the initial investment value). In other
words S = 2P. Manipulate the equation to solve for the variable that you are looking for

Note for question 8 – from the answer above, you can see that “t” is a function of “r”. Substitute the given
investment return rate to find the resulting value of t

Note for question 9 and 10 – Jenny has a total of 90k to invest. There are 2 options for funds that she can
invest in. she therefore needs to split her money between these two investments.
First hint: let x = amount invested in fund 1, and y = amount invested in fund 2. Remember that she cannot
invest more than 90k in total because that is all she has. You have enough information to come up with
the first equation
Second hint: the amount invested in fund 1 (which is equal to x) will earn 10% interest and the amount
invested in fund 2 (which is equal to y) will earn 15% interest. The total amount of interest earned from
both funds is 10k. This information gives you your second equation

You now have 2 equations in 2 variable. Refer to the note simultaneous linear equation in two variables
if you are still unsure on how to solve for this
Follow a similar approach for question 10, remembering that the total volume of liquid from both solution
will be 20ml and that the total concentration of the combined solution needs to be 8%

Note for question 11 and 12 – you have 2 function, a supply function and a demand function. Both these
functions are linear. You have 2 variables, the number of dryer manufactured and the price of the
hairdryer. You need to determine which variable is dependent and which is independent….once you
get this right then the rest is fairly straight forward!

Think of yourself as a consumer who demand a hairdryer – will the amount of hairdryers you want to buy
depend on the price or will the price of hairdryers on the market depend on how many hairdryers you
wish to buy?

The relationship between wholesalers and manufactures is important to note. The manufactures are the
suppliers and the wholesalers demand the product. The part of the text that speaks of wholesalers will
therefore help you formulate the demand function. Similarly, the info on the manufacture will give you
what you need to formulate the supply function. The information you are given is in the form of co-
ordinates…and that is why it’s important to know which variable is dependent (and will be “y”) and
variable is independent (and will be “x”), so that you get the order (x,y) correct

For the demand function you will then get 2 sets of co-ordinates

quantity price
20 210
30 230

You can find the equation for this function. Refer to the note on this study unit if still uncertain on how to
do this

Similarly for the supply function you also get 2 sets of co-ordinates

quantity price
20 210
30 230

You can find the equation for this function. Refer to the note on this study unit if still uncertain on how to
do this
Once you have the two linear equations, you solve as you would any set of simultaneous linear equations

Note for question 13 – this is straight forward manipulation of the equation. Refer to the note on this study
section if you are still unclear on how to solve for this

Note for question 14 and 15 – you have two types of products (coffee maker and cappuccino maker)
being produced in two factories (Atlanta and Fort Worth) every single hour

Total orders of coffee makers will include those made from Atlanta and those made from Fort Worth in
any given hour. You also know that these orders will be at least 64 000. You have been told what to make
“x” and what to make “y”. There is enough information for your first inequality. You now need to decide
if “at least” 64 000 means greater than “>”, greater than and equal to “≥”, less than “<” or less than and
equal to “≤”

Similar thing for the second equation: Total orders of cappuccino makers will include those made from
Atlanta and those made from Fort Worth in any given hour. These orders will be at least 40 000. There is
enough information for your second inequality. You now need to decide if “at least” 64 000 means
greater than “>”, greater than and equal to “≥”, less than “<” or less than and equal to “≤”

Extreme points of the solution space happen where linear functions intersect/cross….in other words,
where they are equal

Note for question 16 - this is similar to question 14, but I will assist with the interpretation of the text
You have 2 types of goods (standard chair and a plush chair). You have two types of services
(construction and upholstering)

2 hours to construct a standard chair and 3 hours for a plush chair. 240 manufacturing hours available
1 hour to upholster a standard chair and 3 hours for a plush chair. 150 upholstering hours available
Construct the linear inequalities based on the information above ad solve for the system simultaneously.
Refer to the examples in the note if you are still uncertain how to do this
Once solved, find the extreme points of the solution space

From the list you should be able to tell which is NOT an extreme point…..as per the question

Note for question 17 and 18 – profits = revenue less expenses


Revenue is the amount of money that “comes into” a business.

If you sell pens at R4 a pen then the revenue from selling 10 pens is (4)*(10) = R40

You have been provided with a cost function, therefore you can find the profit function (simple
subtraction)

To find the marginal profit you need to differentiate the profit function

Substitute the production level to find the marginal profit at that particular level

Refer to the note on this study unit if you are still not comfortable with this concept

Note for question 19 and 20 – same as above

GOOD LUCK!
ASSIGNMENT 2 TIPS

Students often experience challenges with Functions & representation of functions, Linear systems and Application
of differentiation topics of this module. We will explain the principles required to solve questions in these study units
using the examples below. You will also realize that most of the questions will require you to formulate a linear equation, a
system of linear equations or a system of linear inequalities and then solve. I hope the examples will enable you answer
assignment 2 questions with deep understanding of the underlying principles.

Question 1

A children's play ground has just been opened in “Leisure City” and access cards cost R60 per day. In addition, each
activity in the play ground costs 75 cents. Express the amount spent in rand (y) by a child in a day in terms of the number
of activities (x) he/she has taken part in.

Solution

Note that every child must first pay a fixed cost of R60 for the access card.

Secondly if a child takes part in x activities, he/she will have to pay a total of R 0.75x for the activities (we have divided 75
cents by 100 to get R 0.75 before multiplying by x)

Hence the total amount spent by a child in a day is y = 0.75x + 60

Question 2

Electrocars is a company which manufactures electric cars. When the price of petrol increases by 50 cents, the number of
electric cars sold in that month increases by 10. Last month, the price of petrol was R13.20 and the number of electric
cars sold was 300. Let x denotes the price of petrol and y denotes the number of electric cars sold in a month. Express
the number of electric cars (y) sold in a month as a linear function of the price of petrol (x).

Solution

The linear function is of the form y = ax + b, where “a” is the gradient and “b” the y-intercept.

         


The gradient, a= = = 20
        .

Hence the equation now becomes y = 20x + b.

Note that last month, x = 13.2 and y = 300.

Substituting these values into the equation above will enable us to find “b” as follows

300 = 20(13.2) + b

300 = 264 + b

300 – 246 = b

b = 36

Hence, the linear function is y = 20x + 36


Questions 3 and 4 are based on the following information:


Consider the quadratic function y = 2x - x2 and its associated graph


Question 3

Determine the vertex of the graph

Solution
2
If we compare the above equation with the general quadratic function y = ax + bx + c, we can see that

a = -  , b = 2 and c = 0


The x value at the vertex is obtained from x = -


 
Hence x=-  = - 2 (- ) = 3



In order to find the y value at the vertex, we will substitute the x value of the vertex into the quadratic function above as
follows

y = 2(3) - (32) = 3


Hence the coordinates of the vertex are (3, 3)

Question 4

Find the coordinates of the x-intercepts of the graph.

Solution

The x-intercepts are determined by equation the quadratic function to zero and solving for the values of x. Hence

 2 
2x - x = 0 where a = - , b = 2 and c = 0
 

(We can factorize out x in order to solve the equation easily but I will use the quadratic formula to show the general
principle). We can determine the x-intercepts using the quadratic formula as follows

±"  #
=


1
−2 ± &2 − 4 (− + (0)
3
=
1
2(− )
3

−2 ± √4
=
2

3

x = 0 or x = 6

Note that the curve graph cuts the x-axis when y = 0. Hence the coordinates of the x-intercepts are

(0, 0) and (6, 0)


Questions 5 and 6 are based on the following information:

Question 5

Determine the vertex of the graph.

Solution

The vertex here is the coordinates of the minimum turning point of the graph which is (1, -3)

Question 6

Determine the y-intercept of the graph.

Solution

The coordinates of the y-intercept are read from the point where the parabola cuts through the y-axis. In this case, the y-
intercept has coordinates (0, -2).

Questions 7 and 8 are based on the following information:

Pedro invested an amount of money p, in dollars at a continuously compounded interest rate r for time t years.

Question 7

Find the time taken for the amount invested to treble in logarithmic form.

Solution

Present value = P

Future value S = 3P (since the amount trebled)

Rate = r

Using formula for continuous compounding, we have


rt
S=Pe
rt
3P = P e

Cancelling out P gives

3 = ert

Taking logs of both sides give


rt
ln3 = ln e (Ln brings down the exponent “rt” and removes the exponential function e)

ln3 = rt


t=


Question 8

In how many years will the investment trebled if the investment earns 5.5% compounded continuously.

Solution


Substituting the interest rate 5.5% = 0.055 into t = gives the time taken for the investment to trebled.



t=
.

t = 19.97 ̴ 20 years

Question 9

Thato has R 200 000 to invest but in order to diversify his risk exposure, he has chosen two investment options. The first
investment option pays 5% per annum and the second investment option pays 8% per annum. He intends to make an
annual profit of R 12 400. How much will he invest in each investment option?

Solution

The future value of the payments at the end of 1 year must be equal to the value of R 212,400 (200 000 + 12400 interest)
at the end of 1 year.

Let’s assume that an amount ‘P’ is invested at 5% per annum while (200 000 - P) is invested at 8% per annum.
T
Hence, using the formula S = P (1 + R) , we will have:
1 1
212 400 = P (1 + 0.05) + (200 000-P) (1 + 0.08)

212 400 = P (1.05) + (200 000-P) (1.08)

Expanding the brackets give

212 400 = 1.05P + (200 000 x 1.08) -1.08P

212 400 = 1.05P + 216 000 -1.08P

212 400 = 216 000 -0.03P

0.03P = 216 000 – 212 400


0.03P = 3 600

P = 3 600/0.03

P = 120 000

Hence, R120 000 is invested at 5% per annum while R80 000 is invested at 8% per annum.

Question 10

A chemist has two chemical solutions that contain different concentrations. He wants to use these two solutions to
prepare a cleaning chemical that will be effective in removing grease from plastics. One solution has a 16% concentration
and the other has a 7% concentration. How many litres (l) of each solution should he mix to obtain 30 litres (l) of a 10%
concentration.

Solution

Let x = number of litres of 16% concentration

Let y = number of litres of 7% concentration

We also know that he needs to prepare 30 litres of 10% concentration. Hence

16x + 7y = 30 x 10

16x + 7y = 300 (equation 1)

Also the total volume of solution to be prepared is 30 litres. Hence

x + y = 30 (equation 2)

We will then solve the two equations simultaneously as follows:

From equation 2, y = 30 - x

Substituting into equation 1 gives

16x + 7(30 - x) = 300

16x + 210 -7x = 300

16x - 7x = 300 -210

9x = 90
/
x= = 10 litres.
/

But y = 30 - x

y = 30 - 10

y = 20 litres.

Hence we need 10 litres of 16% concentration and 20 litres of 10% concentration in order to prepare the cleaning
chemical.
Questions 11 and 12 are based on the following information:

The University senate has decided to be supplying Apple laptops to staff. A decision has been taken to buy 200 laptops
per year if the price is R4 000 and 150 laptops per year if the price is R5 000. Apple Company is only willing to supply 150
laptops per year if the price is R4 400 and 200 laptops per year if the price is R4 900.

Question 11

Determine the market equilibrium point.

Solution

Market equilibrium is the price at which the quantity demanded is equal to the quantity supplied. We will need to find the
demand equation and supply equation. These equations will be solved simultaneously in order to obtain the price and
quantity at equilibrium point. The demand and supply equation will be in the form p = aq + b where p is the equilibrium
price and q is the quantity at equilibrium, “a” is the gradient and b is the y-intercept.

 # 


For the demand function, a = = = -20
  

The demand equation now becomes p = -20q + b. Substituting p = 4000 and q = 200 into the equation, we can find b as
follows

4000 = -20(200) + b

4000 = -4000 + b

8000 = b

Hence the demand equation is p = -20q + 8000.

#/ ## 


For the supply function, a =  
= 
= 10

The supply equation now becomes p = 10q + b. Substituting p = 4400 and q = 150 into the equation, we can find b as
follows

4400 = 10(150) + b

4400 = 1500 + b

2900 = b

Hence the supply equation is p = 10q + 2900.

We can solve the two equations simultaneously as follows

-20q + 8000 = 10q + 2900

-20q -10q = 2900 – 8000 (collecting like terms)

-30q = -5100

q= =170


But p = -20q + 8000, substituting q = 170 into the equation gives

p = -20(170) + 8000

p = 4600.

Question 12

If Apple is taxed 15% per unit sold, find the new equilibrium point.

Solution

The tax affects the supply function but not the demand function. Hence, Apple will be willing to supply 150 laptops per
year if the price is R5 060 (4400 + (15% of 4400)) and 200 laptops per year if the price is R5 635 (4900 + (15% of 4900)).

0 0 1


For the supply function, a = = = 11.5
  

The supply equation now becomes p = 11.5q + b. Substituting p = 5060 and q = 150 into the equation, we can find b as
follows

5060 = 11.5(150) + b

5060 = 1725 + b

3335 = b

Hence the supply equation is p = 11.5q + 3335.

We can solve the two equations simultaneously as follows

-20q + 8000 = 11.5q + 3335

-20q -11.5q = 3335 – 8000 (collecting like terms)

-31.5q = -4665

#00
q= = 148
.

But p = -20q + 8000, substituting q = 148 into the equation gives

p = -20(148) + 8000

p = 5040.
Question 13

Solve the inequality -27 + 5x > 3(8 +7x)

Solution

-27 + 5x > 24 + 21x (expanding the brackets on the right)

5x - 21x > 24 + 27 (collecting like terms together give)

Hence

-17x > 51

Dividing all through by the -17 will reverse the sign of the inequality and give

x < -3

Questions 14, 15 and 16 are based on the following information:

A furniture manufacturer makes two types of furniture – chairs and sofas. The production of the sofas and chairs requires
three operations – carpentry, finishing, and upholstery. Manufacturing a chair requires 3 hours of carpentry, 9 hours of
finishing, and 2 hours of upholstery. Manufacturing a sofa requires 2 hours of carpentry, 4 hours of finishing, and 10 hours
of upholstery. The factory has allocated at most 66 labour hours for carpentry, 180 labour hours for finishing, and 200
labour hours for upholstery. The profit per chair is R90 and the profit per sofa is R75. How many chairs and how many
sofas should be produced each day to maximize the profit? Let x be the number of chairs produced and y be the number
of sofas produced.

Question 14

Write the system of inequalities that describe the furniture manufacturer’s constraints.

Solution

3x + 2y ≤ 66 (carpentry hours constraint)

9x + 4y ≤ 180 (finishing hours constraint)

2x + 10y ≤ 200 (upholstery hours constraint)

X ≥ 0, y ≥ 0 (number of chairs and sofas produced must be greater than


zero)

Question 15

How many chairs and how many sofas should be produced each day to maximize the profit?

Solution

This can be obtained by solving the system of equations. The easiest way is to plot these inequalities and determine the
region that satisfies the inequalities.
We will use the points of intersection to determine the number of chairs and sofas that will produce the maximum profit

p = 90x + 75y as follows;

When x = 0 and y = 0, p = 90(0) + 75(0) = 0

When x = 0 and y = 20, p = 90(0) + 75(20) = 1500

When x = 10 and y =18, p = 90(10) + 75(18) = 2250

When x = 16 and y = 9, p = 90(16) + 75(9) = 2115

When x = 20 and y = 0, p = 90(20) + 75(0) = 1800

The manufacturer must produce 10 chairs and 18 sofas in order to maximize the profit.

Question 16

What is the maximum profit realized in a day?

Solution

The maximum profit is R2250 as per the calculation above.

Questions 17 and 18 are based on the following information:

A Company specializes in the production of washing machines. The cost in rand to produce a washing machine is
C(x) = 300
and the revenue in rand obtained from selling a washing machine is
2
R(x) = 800 – x + 0.05x
Where x is the number of washing machines produced.
Question 17

Determine the marginal profit function

Solution

The cost in rand to produce x washing machines is 300x.

2 3
The revenue from selling x washing machines is 800x – x + 0.05x

First, find the profit function by subtracting the cost function from the revenue function.
2 3
p(x) = (800x – x + 0.05x ) – 300x
3 2
p(x) = 0.05x – x + 500x
Then, determine the marginal profit function by differentiating the profit function as follows
1 2
p (x) = 0.15x - 2x + 500

Question 18

What is the marginal profit obtained by selling 50 washing machines?

Solution

The marginal profit is obtained by substituting 50 into the marginal profit function as follows

1 2
p (x) = 0.15x - 2x + 500

1 2
p (50) = 0.15(50) - 2(50) + 500

1
p (50) = 775

Questions 19 and 20 are based on the following information:

The cost in rand for the daily production of x kilograms of gold by a mining company is given by the function
 2
C(x) = - x + 200x + 3000.


Question 19

Determine the marginal cost to produce these kilograms of gold.

Solution

The marginal cost function is obtained by differentiating the cost function as follows:

C (x) = - (2) x + 200
1

1 
C (x) = - x + 200


Question 20

Calculate the marginal cost to produce 150 kilograms of gold.


Solution

The marginal cost (MC) to produce 150 kilograms of gold is obtained by substituting 150 into the marginal cost function as
follows


C1(150) = - (150) + 200

1
C (150) = 194
ASSIGNMENT 03 EXAMPLES (PART 2)

QUESTION 8

An investment of R1 000 was made at the end of each month by Nelson for 6 years in an account
that earns 11% compounded annually. After the first 6 years, Nelson stopped investing but left the
money in the account for the next 25 years to earn 11% compounded annually. How much will
Nelson have at the end of the 25 years?

Solution

> We have to first calculate the future value for an annuity of R1 000 for 6 years. After that we will use
that future value as a new present value for the next 25 years, to find a new future value.

For the calculation of the future value of an annuity, the following information is given:

R = 1 000
0.11
i = 1
= 0.11

n = 6 × 1 = 6 years

S = ?

The future value is calculated as:


(1+𝑖)𝑛 −1
S= R×[ ]
𝑖

(1+0.11)6 −1
= 1 000 × [ ]
0.11

(1.11)6 −1
= 1 000 × [ 0.11
]

= 7 912.86

This future value now becomes the present value for the next 25 years, hence we have to compute a
new future value.

P = 7 912.86
0.11
R= 1
= 0.11 (because it is compounded annually)

T = 25 × 1 = 25 years

S = ?

S = P (1 + R)T

S = 7 912.86 (1+ 0.11)25

S = 107 499.87

The NEW future value is R107 499.87


Using the SHARP financial calculator:

2ndf CA 2ndf I/Y 1 ENT ON/C

----------------------------------------------------

± 1 000 PMT

6 2ndF N N

11 I/Y

---------------------------------------

COMP FV 7 912.86 is displayed on the screen

This future value now becomes the present value for the next 25 years, hence we have to compute a
new future value.

Using the SHARP financial calculator:

2ndf CA 2ndf I/Y 1 ENT ON/C

----------------------------------------------------

± 7 912.86 PV

25 2ndF N N

11 I/Y

---------------------------------------

COMP FV 107 499.87 is displayed on the screen

==================================================================================

QUESTION 9

Mondekazi has decided to deposit R200 at the end of every month for the next 5 years in a bank
account. What will be the future value if the bank pays 6% compounded monthly?

Solution

For the calculation of the future value of an annuity, the following information is given:

R = 200
0.06
i = 12
= 0.005

n = 5 × 12 = 60 months

S = ?

The future value is calculated as:


(1+𝑖)𝑛 −1
S= R×[ ]
𝑖
(1+0.005)60 −1
= 200 × [ 0.005
]

(1.005)60 −1
= 200 × [ 0.005
]

= 13 954.01

The future value of the investment is R13 954.01

Using the SHARP financial calculator:

2ndf CA 2ndf I/Y 12 ENT ON/C

----------------------------------------------------

± 200 PMT

6 I/Y

5 2ndf N N

---------------------------------------

COMP FV 13 954.01 is displayed on the screen

The future value of the investment is R13 954.01

==================================================================================

QUESTION 10

A school need a new school bus that they anticipate will cost R700 000. They decided to save money
for it by investing R7 000 at the end of each month in an account that earns 9% compounded
monthly. How long will it take the school to achieve its goal? (Your answer should be rounded off to
the nearest integer)

Solution

> How long means we are looking for the term or time, n, for an annuity. The following information is
given:

R = 7 000

S = 700 000
0.09
i = 12
= 0.0075

n = ?

The time is calculated as:


(1+𝑖)𝑛 −1
S= R×[ 𝑖
]

(1+0.0075)𝑛 −1
700 000 = 7 000 × [ 0.0075
]

700 000 (1.0075)𝑛 −1


7 000
= 0.0075
(1.0075)𝑛 −1
100 =
0.0075

100 × 0.0075 = (1.0075)𝑛 - 1

0.75 + 1 = (1.0075)𝑛

1.75 = (1.0075)𝑛

log 1.75 = n log 1.0075


log 1.75
n = log 1.0075

n = 74.8948

n ≈ 75

It will take 75 months

Using the SHARP financial calculator:

2ndf CA 2ndf I/Y 12 ENT ON/C

----------------------------------------------------

± 7 000 PMT

700 000 FV (NOTE: we don’t enter ± in front of the second amount if there are two amounts)

9 I/Y

---------------------------------------

COMP N 74.8948, is displayed on the screen

74.8948 ≈ 75

===============================================================================

QUESTION 11

Felix was introduced to an annuity policy by a friend. The policy pays 9%, compounded monthly.
How much should Felix invest now if he wants to be paid R1 200 at the end of each month for the
next 10 years?

Solution

For the calculation of the present value (lump sum) of an annuity, the following information is given:

R = 1 200
0.09
i = 12
= 0.0075

n = 10 × 12 = 120 months

P = ?

The present value is calculated as:


(1+𝑖)𝑛 −1
P=R×[ ]
𝑖(1+𝑖)𝑛

(1+0.0075)120 −1
= 1 200 × [ ]
0.0075(1+0.0075)120

(1.0075)120 −1
= 1 200 × [ 0.0075(1.0075)120
]

= 98 905.78

The present value is R98 905.78

Using the SHARP financial calculator:

2ndf CA 2ndf I/Y 12 ENT ON/C

----------------------------------------------------

± 1 200 PMT

10 2ndF N N

8 I/Y

---------------------------------------

COMP PV 98 905.78 is displayed on the screen

=================================================================================

QUESTION 12

What is the present value of an annuity that pays R1 600 at the end of every 3-months period for 5.5
years and money is worth 5% compounded quarterly?

Solution

For the calculation of the present value (lump sum) of an annuity, the following information is given:

R = 1 600
0.05
i = 4
= 0.0125

n = 5.5 × 4 = 22 quarters

P = ?

The present value is calculated as:


(1+𝑖)𝑛 −1
P=R×[ 𝑖(1+𝑖)𝑛
]

(1+0.0125)22 −1
= 1 600 × [ 0.0125(1+0.0125)22
]

(1.0125)22 −1
= 1 600 × [ 0.0125(1.0125)22
]

= 30 608.90

The future value is R30 608.90


Using the SHARP financial calculator:

2ndf CA 2ndf I/Y 4 ENT ON/C

----------------------------------------------------

± 1 600 PMT

5.5 2ndF N N

5 I/Y

---------------------------------------

COMP PV 30 608.90 is displayed on the screen

==================================================================================

QUESTION 13

Determine the lump sum required to generate payments of R3 000 at the end of each half-year for 4
years if money is worth 6% compounded half-yearly.

Solution

For the calculation of the present value (lump sum) of an annuity, the following information is given:

R = 3 000
0.06
i = 2
= 0.03 (We dividing by 2 because it is compounded half-yearly)

n = 4 × 2 = 8 half years (We multiplying by 2 because it is compounded half-yearly)

P = ?

The present value is calculated as:


(1+𝑖)𝑛 −1
P=R×[ 𝑖(1+𝑖)𝑛
]

(1+0.03)8 −1
= 3 000 × [ 0.03(1+0.03)8
]

(1.03)8 −1
= 3 000 × [ 0.03(1.03)8
]

= 21 059.08

The lump sum is R21 059.08

Using the SHARP financial calculator:

2ndf CA 2ndf I/Y 2 ENT ON/C

----------------------------------------------------

± 3 000 PMT

4 2ndF N N
6 I/Y

COMP PV 21 059.08 is displayed on the screen

=================================================================================

QUESTION 14

Frank has decide to purchase a second house for rentals, and can afford to pay R2 500 as monthly
mortgage. What is the most expensive house that Frank can purchase if the bank offers him a loan
for 30 years at 10.5% compounded monthly with the condition that he makes an initial down
payment of R50 000?

Solution

> We first calculate the present value of the annuity, then we add the down payment (deposit).

For the calculation of the present value of an annuity, the following information is given:

R = 2 500
0.105
i = 12
= 0.00875

n = 30 × 12 = 360 months

P = ?

The present value is calculated as:


(1+𝑖)𝑛 −1
P=R×[ ]
𝑖(1+𝑖)𝑛

(1+0.00875)360 −1
= 2 500 × [ ]
0.00875(1+0.00875)360

(1.00875)360 −1
= 2 500 × [ ]
0.00875(1.00875)360

= 273 301.91

We then add the down payment: 273 301.91 + 50 000 = 323 301.91

Frank can purchase a house worth R323 301.91

Using the SHARP financial calculator:

2ndf CA 2ndf I/Y 12 ENT ON/C

± 2 500 PMT

10.5 I/Y

30 2ndf N N

COMP PV 273 301.91 is displayed on the screen

273 301.91 + 50 000 = 323 301.91

Frank can purchase a house worth R323 301.91


Simple interest

For all your simple interest calculations, you must remember your formulas:

 I = PRT
 S=P+I
 S = P(1 + RT)

where:

 I = simple interest amount


 P = present value
 R = simple interest rate
 T = term

Step to follow:
 Read the question carefully and note that the given information as you go
 Plug the given info into the formula and solve for the unknown variable,
noting that you may need to manipulate the equation as required
 Interest rate is typically given as a percentage. Always divide this by 100 in
your calculation
 Similarly, when you are solving for “R”, always remember to multiply your final
answer by 100 to get to the interest rate

NB note:
 The units used for the interest rate have to be the same as the units used for
the term. In other words, if the term is months and the interest is charged per
annum you have to adjust to a common unit before you can go any further
with your calculation
 Refer to example 7.1 in your study guide where you have a simple interest
rate per annum but the term of the loan is 90 day. In this case you need to
convert the 90 days into a yearly equivalent. Given that there are 365 days in
90
a year, a 90 day period is the same as 365 (i.e. the term is 90 days out of a
total of 365 days). If in the same example, the term of the loan was a quarter
of a year then ask yourself: how many quarters are there in year….the answer
1
is “4”, and so this would be equivalent to 4 (i.e. the term is one out of a total of
4 quarters)

Following these guidelines, you should be able to answer questions 1, 2 and 3 of the
assignment

Let’s go through some examples:

Example 1 – solving for the interest amount


Martha invested R1 500 for 21 months. The applicable simple interest rate is 9% per
year. Determine the amount of interest that Martha will receive in 21 months’ time

Solution:
P = 1 500
21
T = 21 months (= 12 months since there are 12 months in a year)
R = 9% per annum
You need to solve for I
 I = PRT
9 21
 I = (1 500)*( )*( ) = 236.25
100 12
Therefore Martha will receive R236.25 in interest at the end of the period

Note how the term of 21 months is “treated” (i.e. diving by 12 since there are 12
months in a year). When the period is quoted in terms of months, you would do the
same whether the period is less than or greater than a year…as is the case in Q3 of
the assignment.

Example 2 – solving for the principal amount


Sibongile must pay Jabu an amount of R3 500 in 6 years’ time. A 12.5% simple interest
rate is applicable. Determine the amount that Sibongile borrowed from Jabu 6 years
ago

Solution:
S = 3 500
T = 6 years
R = 12.5% per annum

You need to solve for P

 S = P(1 + RT)
12.5
 3 500 = P(1 + * 6)
100
 3 500 = P(1.75)
𝟑 𝟓𝟎𝟎
 =P
𝟏.𝟕𝟓
 2 000 = P

Therefore Sibongile borrowed R2 000 from Jabu six years ago

Example 3 – solving for the interest rate


Temba borrowed R12 500 from Thabo. He must pay him R20 000 six years from now.
Determine the applicable simple interest rate at which Temba borrowed the money

Solution:
P = 12 500
S = 20 000
T = 6 years

You need to solve for R

 S = P(1 + RT)
 20 000 = 12 500(1 +R*6)
20 000
 = 1 + R*6
12 500
 𝟏. 𝟔 = 1 +R*6
 1.6 - 1 = R*6
 0.6 = R*6
0.6
 =R
6
 0.1 = R
REMEMBER to multiply this by 100 to give R = 10%

Therefore the applicable simple interest rate is 10% per annum

NB note – solving for the interest amount is not the same thing as solving for the
interest rate. Always read the question carefully, and use the appropriate formula!!

This note is relevant for Q4 of the assignment, even though it is a compound interest
question!

Compound interest

The trick to these calculations is working out the effective interest rate per annum.
Once this known, then you can simply plug the given information into the formula
and solve for the required variable.

 S = P(1 + R)T
R in the equation is the effective interest rate per annum

Once you get an interest that is compounded in over anything other than “once per
year”, then the first step is to find the equivalent of R

Refer to the first question of exercise 7.3 in the study guide:


“Determine the interest earned if R1 000 is invested for one year at 8% per annum.
But the interest rate is compounded biannually”

So instead of R (which is an interest rate per annum that is compounded once a


year), we have a rate of interest that is compounded every six months

Ask yourself: how many “six months” are there in a year…the answer should be “2”

8%
So this means that the effective interest rate for a six month period is
2
Because there are two opportunities to earn interest over the year, your term is
“extended” from 1 year to 2 periods (this is effectively your new “T”)

So remember this:
 If interest is compounded annually, that is the effective interest rate that is
paid at the end of each year.
 If interest is compounded semi-annually (half-yearly), the quoted interest rate
is divided by 2 in order to get the effective interest rate paid after every 6
months.
 If interest is compounded quarterly (4 quarters in a year), the quoted interest
rate is divided by 4 in order to get the effective interest rate paid after every 3
months.
 If interest is compounded monthly (12 months in a year), the quoted interest
rate is divided by 12 in order to get the effective interest rate paid after every
month.

In each of these cases, you need to extend the term accordingly!

Let’s look at an example:


Suppose you have 5 numbers of the LOTTO Powerball and your share of the winnings
amounts to R200 000. You decide to invest this for your child’s education, who will be
going to university in 6 years. The fund you chose offers interest at a rate of 12% per
annum compounded quarterly. What will the value of your investment be at this
time?

 At what rate is the compounding happening?.....4 times a year


(compounded quarterly)
12% 0.12
 What is the effective interest rate per period (i.e. per quarter)?….. = =
4 4
0.03 = 3%
 Over the 6 year period, how many times will the interest be due/paid?......4
periods in a year for 6 years gives you a total of 24 periods
 So you have S = P(1 + 3%)24 = R406 558.82…..you know what “P” is so you can
work this out on your calculator

You should now be able to do question 5, 6 and 7 of the assignment

Note: the variable you are solving for question 7 is in the exponent…refer to study
unit 4.4 if you have forgotten how to solve for these.

Time Value of Money

As we progress into the Mathematics of Finance study unit you will note that later
sections build on the knowledge of the sections before it. It is therefore essential that
you have a good understanding of compound interest before attempting the
exercises on the time value of money.

Basic Formula is S = P(1+R)T

Interpretation of the equation is very important. To find the future value at the end of
the period (S), we need to accumulate the principal amount at the given rate for
the applicable period. In other words the accumulating factor is (1 + R)T

S
If we manipulate to make P the subject of the formula, then we get: P =
(1+R)T

In other words, to find the present value (the original principal amount), we need to
1
discount the accumulated value S, to the present time. And therefore is the
(1+R)T
discounting factor.
In mathematical terms, accumulating and discounting are inverse functions, and
1
hence the discounting factor =
accumulating factor

NB Tip
 Drawing a timeline will be very useful. Placing your numbers correctly on the
timeline from the given information is a critical first step. If drawn properly, this
will also show you the point to which you need to accumulate or discount.
 When drawing your timeline remember to use time periods that match the
compounding period – for ease of reference

Please go through the examples in the study guide as well as the exercises in the
workbook (worksheet 3 on page 151).
Further to this, the concept learnt here will be used in the annuities section which
follows, so make sure that you are comfortable with this concepts before you move
on.

Annuities

An annuity is a set of regular payments made or received over a specified term.

 Ordinary annuity - payments are made in arrears

 Annuity due – payments are in advance

 Annuity certain – payments are for a fixed term

 Perpetuity – payments go on forever (no fixed term)

These distinctions will be important for your assignment. Note for instance that Q8 of
assignment 3 speaks to a R2 000 payment made at the end of the year. This is an
ordinary annuity.

Make sure that you read the question carefully and use the correct formula in your
assignment and in your exam!!

For continuous compounding, the accumulated value can be found by:


S = PeRT
You should be able to manipulate this equation to find any of the other variables.

NB note for annuities:

 Term and interest rate are no longer represented by R and T respectively


 n = number of periods
 i = interest rate
 R = periodic payment
Let’s look at some examples:

Example 4 – solving for the future value


Determine the accumulated amount of an annuity with quarterly payments of
R1 200, at an interest rate of 16% per annum compounded quarterly, made over a
period of 10 years.

Solution:

The formula for future value is


It is important that these are
 Period over which payments are made = quarterly the same!
 Period over which interest is compounded = quarterly

So we know the following:

 R = 1 200 (periodic payment)


0.16
 i = 4 = 0.04 (effective interest rate per quarter)
 n = 10*4 = 40 quarters (10 years, 4 quarters per year)
(1.04)40 −1
 S = (1 200)∗ ( 0.04
)
 S = 114 030.62

Example 5 – solving for the term


Mokgadi needs R55 000 for her dream holiday to Mauritius to celebrate her 30th
birthday in a few years’ time? She plans to save for this by making annual deposits of
R9 500 into an investment account that earns interest at 7.34% interest per year. If
Mokgadi’s birthday is in exactly 4 years time, will she have saved enough for her
goal?

Solution – you need to find out how long it will take Mokgadi to accumulate R55 000,
and then compare this to the time frame that she has

S = 55 000

R = 9 500

i = 7.34%

note that interest is compounded once a year and the investment payments are
also made once a year…so no adjustments necessary

1.0734n −1
55 000 = (9 500)*( )
0.0734
55 000 1.0734n −1
=
9 500 0.0734
1.0734n −1
5.789474 =
0.0734

(5.789474)*(0.0734) = 1.0734n - 1

0.424947+ 1 = 1.0734n

1.424947 = 1.0734n

The variable that you wish to solve for is in the exponent. We therefore what you
learnt in study unit 4 on log and exponential functions!

Taking the natural log of both sides gives:

ln(1.424947) = n *ln(1.0734)
0.35413 = n * (0.070831)
0.35413
=n
0.070831
4.9997 = n

It will therefore take Mokgadi close to 5 years to accumulate the required R55 000.
Given that her birthday is in exactly 4 years’ time, she will not fulfil this goal

Example 6 – solving for the periodic payment


Senzeni owes Petrus an amount of R120 000. This debt must be paid off in 6 years’
time. An interest rate of 13.6% per annum compounded quarterly is applicable.
Determine Senzeni’s quarterly payments

Solution:

The formula for present value is

So we know the following:

 P = 120 000 (loan amount)


0.136
 i= = 0.034 (effective interest rate per quarter)
4
 n = 6*4 = 24 quarters (6 years, 4 quarters per year)

So, (1 +1)n = (1.034)24 = 2.230966


2.230966−1 1.230966
Plugging into the formula: 120 000 = R (
(0.034)∗(2.230966)
)=R( ) = R(16.22834)
0.075853
120 000
=R
16.22834

7 394.47 = R

You should now be able to do up to Q13 of the assignment

Hint for Q8

 draw a timeline to represent all the information you have been provided
 break the question up into 2 parts
o the first part is a regular annuity over a period of 8 years. Find the future
value
o the second part an investment of a lumpsum (there are no regular
payments) over a 36 year period

Amortisation

Key concepts to keep in mind:

 every instalment represents a repayment of both capital and interest


 the interest portion within each instalment reduces with time
 the balance outstanding at any given point is the present value of all future
instalments at that point in time

Working out the present value of a loan as required in Q14 and Q16 is similar to
solving annuity type question as done above

If you need a loan to buy a house or a car and you have a deposit, then the
amount of money to be borrowed equals the value of the house/car less the deposit
that you put down. Keep this in mind!

Example 6 – amortization schedule


Draw up an amortization schedule for a loan of R12 000 with interest at 10% per
annum compounded yearly over a term of five years.

P = 12 000
i = 0.1
n=5
You need to solve for R (annual payments, in this instance)

1.15 −1
12 000 = R ( )
(0.1)∗(1.1)5
0.061051
12 000 = R ( )
0.161051
12 000 = R(3.790787)
12 000
=R
3.790787
3 165.57 = R

This is similar to what is being asked of you in Q15 and Q18 of the assignment

Every instalment covers a portion of capital and a portion of interest, as stated


above. You now need to determine how much interest is due in the first year:
I = PRT……where T = 1 (don’t consider the whole term of the loan, since you are
interested in the interest due in the first year)
I = (12 000)*(0.1)*(1) = 1 200

Therefore, out of the yearly instalment of R3 165.57, R1 200 of that will go to interest
payment and the rest will go towards the principal repayment. Your debt after this
first instalment will therefore be reduced by R1 965.57 (3 165.57 – 1 200)

At the start of the second year, therefore, the amount outstanding (the new loan
value) is R10 034.43

Remember that interest is due on the amount outstanding. Therefore as you pay off
the loan over time the balance outstanding also reduces. Since your instalment is
constant, this means that more of less of the instalment goes to cover interest and
more goes to paying off the principal debt. This can be seen from the table clearly
 in the first year the interest due was R1200 (10% of R12 000), whereas the
interest due in the second year wasR1 003.44 (10% of R10 034.43)
 in the second year the outstanding loan balance is R10 034 (R12 000 less
R1 965.57), since some capital has been repaid. The interest amount is
therefore reduced (10% of a smaller amount), and therefore more of the
instalment is sued to repay the principal debt
 …and so on

Outstanding Periodic Interest due in Principal repaid after


balance Instalment that period instalment
At the start 12 000 3 165.57 1 200 1 965.57
of year 1 (original loan (constant (difference between
amount) throughout term) instalment and
interest amount)
At the start 10 034.43 3 165.57 1 003.44 2 162.13
of year 2 (constant (10% of (difference between
throughout term) R10 034.43) instalment and
interest amount)
At the start 7 872.30 3 165.57 787.23 2 378.34
of year 3 (constant (10% of (difference between
throughout term) R7 872.30) instalment and
interest amount)
At the start 5 493.96 3 165.57 549.40 2 616.17
of year 4 (constant (10% of (difference between
throughout term) R5 493.96) instalment and
interest amount)
At the start 2 877.79 3 165.57 287.78 2 877.79
of year 5 (constant (10% of (difference between
throughout term) R2 877.79) instalment and
interest amount)

From the table you are able to read off the balance outstanding after any number
of installments….similar to what is asked in Q17
You can also read off the total amount of interest paid…similar to what is asked in
Q20

These questions can also be answered by using your calculator, without having to
draw up an amortization schedule. Please consult the calculator manual and ensure
that you are comfortable with these calculations

Powered by TCPDF (www.tcpdf.org)

You might also like